Обратный ход гаусса: Метод Гаусса

Содержание

Обратный ход метода Гаусса

Запишем линейную систему, получившуюся в результате прямого хода метода Гаусса, с расширенной матрицей, приведенной в табл. 3.3.:

(3.2.10)

Система (3.2.10) равносильна исходной системе (3.2.1) и ее решение получить несложно. Из последнего уравнения системы можно сразу найти , из предпоследнего , и т. д.

Алгоритм и расчетные формулы для обратного хода метода Гаусса

  1. Приближенная проверка невырожденности матрицы системы. Если , то выдать сигнал о том, что матрица системы близка к вырожденной и закончить решение системы. При выполнении условиямодуль определителя матрицы системы будет меньшеи близок к нулю.

  2. Вычисление решения по рекуррентным формулам

, (3.2.11)

, . (3.2.12)

На этом можно было бы завершить алгоритмическую схему метода Гаусса с выбором главных элементов в столбцах. Осталось только сделать последнее замечание. В процессе обратного хода метода Гаусса используются только элементы матрицы, стоящие выше главной диагонали и . Нули, стоящие под главной диагональю, и единицы, стоящие на главной диагонали, для получения решения не нужны. Поэтому их можно не вычислять. А для того чтобы не делать эти лишние операции, можно

исключить из алгоритма прямого хода вычисления по формулам (3.2.6), (3.2.8) при .

Применение метода Гаусса для вычисления определителей Теоретические основы

При вычислении определителя квадратной матрицы А можно использовать прямой ход метода Гаусса. Применение метода Гаусса для вычисления определителей основано на их свойствах:

  1. Если поменять местами две строки матрицы, то ее определитель поменяет свой знак, а абсолютная величина определителя не изменится .

  2. Если какую-либо строку матрицы разделить на некоторую постоянную с, не равную нулю, то определитель матрицы уменьшится в с раз .

  3. Если из какой-либо строки матрицы вычесть другую строку, умноженную на некоторое число, не равное нулю, то определитель не изменится .

  4. Если матрица является верхней треугольной (все ее элементы, лежащие ниже главной диагонали, равны 0), то её определитель будет равен произведению диагональных элементов этой матрицы.

В результате преобразований прямого хода метода Гаусса матрица

А превратится в верхнюю треугольную матрицу . На главной диагонали матрицыбудут стоять единицы на всех строчках, кроме последней. А в последней строке на главной диагонали будет стоять . Поэтому определитель преобразованной матрицы будет равен (произведению диагональных элементов). В процессе прямого хода метода Гаусса используются перестановки строк, меняющие знак определителя, но не меняющие его модуль. Обозначим через s число перестановок строк, совершаемых в процессе прямого хода. Кроме того, в процессе прямого хода производятся деления строк матрицы на элементы , которые приводят к тому, что величина определителя будет разделена на произведение. Причемздесь используются значения этих величин после перестановки строк, если она производится. Таким образом, . Отсюда

. (3.2.13)

Все величины, входящие в эту формулу, кроме s, вычисляются в процессе прямого хода метода Гаусса, а вычислить величину s не составляет труда.

Обратный ход метода Гаусса — Мегаобучалка

Учреждение образования «Белорусская государственная

Сельскохозяйственная академия»

 

 

 

Кафедра высшей математики

 

Методические указания

по изучению темы «Метод Гаусса решения систем линейных

уравнений» студентами бухгалтерского факультета заочной формы получения образования (НИСПО)

 

Горки, 2013

 

Метод Гаусса решения систем линейных уравнений

Эквивалентные системы уравнений

 

Две системы линейных уравнений называются эквивалентными, если каждое решение одной из них является решением другой. Процесс решения системы линейных уравнений состоит в последовательном преобразовании её в эквивалентную систему с помощью так называемых элементарных преобразований, которыми являются:

1) перестановка любых двух уравнений системы;

2) умножение обеих частей любого уравнения системы на отличное от нуля число;

3) прибавление к любому уравнению другого уравнения, умноженного на любое число;

4) вычёркивание уравнения, состоящего из нулей, т.е. уравнения вида .

 

Гауссовы исключения

 

Рассмотрим систему m линейных уравнений с n неизвестными:

Суть метода Гаусса или метода последовательного исключения неизвестных состоит в следующем.

Вначале с помощью элементарных преобразований исключается неизвестная из всех уравнений системы, кроме первого. Такие преобразования системы называются шагом гауссового исключения. Неизвестная называется разрешающей переменной

на первом шаге преобразований. Коэффициент называется разрешающим коэффициентом, первое уравнение называется разрешающим уравнением, а столбец коэффициентов при разрешающим столбцом.

При выполнении одного шага гауссового исключения нужно пользоваться следующими правилами:

1) коэффициенты и свободный член разрешающего уравнения остаются неизменными;

2) коэффициенты разрешающего столбца, расположенные ниже разрешающего коэффициента, обращаются в нули;

3) все прочие коэффициенты и свободные члены при выполнении первого шага вычисляются по правилу прямоугольника:



, где

i=2,3,…,m; j=2,3,…,n.

Аналогичные преобразования выполним и над вторым уравнением системы. Это приведёт к системе, у которой во всех уравнениях, кроме первых двух, будет исключена неизвестная . В результате таких преобразований над каждым из уравнений системы (прямой ход метода Гаусса) исходная система приводится к эквивалентной ей ступенчатой системе одного из следующих видов.

 

Обратный ход метода Гаусса

 

Ступенчатая система

имеет треугольный вид и все (i=1,2,…,n). Такая система имеет единственное решение. Неизвестные определяются, начиная с последнего уравнения (обратный ход метода Гаусса).

Ступенчатая система имеет вид

где , т.е. число уравнений системы меньше либо равно числу неизвестных. Эта система не имеет решений, так как последнее уравнение не будет выполняться ни при каких значениях переменной .

Ступенчатая система вида

имеет бесчисленное множество решений. Из последнего уравнения неизвестная выражается через неизвестные . Затем в предпоследнее уравнение вместо неизвестной подставляется её выражение через неизвестные . Продолжая обратный ход метода Гаусса, неизвестные можно выразить через неизвестные . В этом случае неизвестные называются свободными и могут принимать любые значения, а неизвестные базисными.

При практическом решении систем удобно выполнять все преобразования не с системой уравнений, а с расширенной матрицей системы, состоящей из коэффициентов при неизвестных и столбца свободных членов.

Пример 1. Решить систему уравнений

Решение. Составим расширенную матрицу системы и выполним элементарные преобразования:

.

В расширенной матрице системы число 3 (оно выделено) является разрешающим коэффициентом, первая строка является разрешающей строкой, а первый столбец – разрешающим столбцом. При переходе к следующей матрице разрешающая строка не изменяется, все элементы разрешающего столбца ниже разрешающего элемента заменяются нулями. А все другие элементы матрицы пересчитываются по правилу четырёхугольника. Вместо элемента 4 во второй строке запишем , вместо элемента -3 во второй строке будет записано и т.д. Таким образом, будет получена вторая матрица. У этой матрицы разрешающим элементом будет число 18 во второй строке. Для формирования следующей (третьей матрицы) вторую строку оставляем без изменения, в столбце под разрешающим элементом запишем нуль и пересчитаем оставшиеся два элемента: вместо числа 1 запишем , а вместо числа 16 запишем .

В результате исходная система свелась к эквивалентной системе

Из третьего уравнения находим . Подставим это значение во второе уравнение: y=3. В первое уравнение подставим найденные значения

y и z: , x=2.

Таким образом, решением данной системы уравнений является x=2, y=3, .

Пример 2. Решить систему уравнений

Решение. Выполним элементарные преобразования над расширенной матрицей системы:

Во второй матрице каждый элемент третьей строки разделили на 2.

В четвёртой матрице каждый элемент третьей и четвёртой строки разделили на 11.

. Полученная матрица соответствует системе уравнений

Решая данную систему, найдём , , .

Пример 3. Решить систему уравнений

Решение

. Запишем расширенную матрицу системы и выполним элементарные преобразования:

.

Во второй матрице каждый элемент второй, третьей и четвёртой строк разделили на 7.

В результате получена система уравнений

эквивалентная исходной.

Так как уравнений на два меньше, чем неизвестных, то из второго уравнения . Подставим выражение для в первое уравнение: , .

Таким образом, формулы дают общее решение данной системы уравнений. Неизвестные и являются свободными и могут принимать любые значения.

Пусть, например, Тогда и . Решение является одним из частных решений системы, которых бесчисленное множество.

 

Вопросы для самоконтроля знаний

 

1) Какие преобразования линейных систем называются элементарными?

2) Какие преобразования системы называются шагом гауссова исключения?

3) Что такое разрешающая переменная, разрешающий коэффициент, разрешающий столбец?

4) Какими правилами нужно пользоваться при выполнении одного шага гауссова исключения?

 

 

Метод гаусса приклад. Обратный ход метода гаусса

Метод Гаусса – это просто! Почему? Известный немецкий математик Иоганн Карл Фридрих Гаусс еще при жизни получил признание величайшего математика всех времен, гения и даже прозвище «короля математики». А всё гениальное, как известно – просто! Кстати, на деньги попадают не только лохи, но еще и гении – портрет Гаусса красовался на купюре в 10 дойчмарок (до введения евро), и до сих пор Гаусс загадочно улыбается немцам с обычных почтовых марок.

Метод Гаусса прост тем, что для его освоения ДОСТАТОЧНО ЗНАНИЙ ПЯТИКЛАССНИКА. Необходимо уметь складывать и умножать! Не случайно метод последовательного исключения неизвестных преподаватели часто рассматривают на школьных математических факультативах. Парадокс, но у студентов метод Гаусса вызывает наибольшие сложности. Ничего удивительного – всё дело в методике, и я постараюсь в доступной форме рассказать об алгоритме метода.

Сначала немного систематизируем знания о системах линейных уравнений. Система линейных уравнений может:

1) Иметь единственное решение.
2) Иметь бесконечно много решений.
3) Не иметь решений (быть несовместной ).

Метод Гаусса – наиболее мощный и универсальный инструмент для нахождения решения любой системы линейных уравнений. Как мы помним, правило Крамера и матричный метод непригодны в тех случаях, когда система имеет бесконечно много решений или несовместна. А метод последовательного исключения неизвестных в любом случае приведет нас к ответу! На данном уроке мы опять рассмотрим метод Гаусса для случая №1 (единственное решение системы), под ситуации пунктов №№2-3 отведена статья . Замечу, что сам алгоритм метода во всех трёх случаях работает одинаково.

Вернемся к простейшей системе с урока Как решить систему линейных уравнений?
и решим ее методом Гаусса.

На первом этапе нужно записать расширенную матрицу системы :
. По какому принципу записаны коэффициенты, думаю, всем видно. Вертикальная черта внутри матрицы не несёт никакого математического смысла – это просто отчеркивание для удобства оформления.

Справка : рекомендую запомнить термины линейной алгебры. Матрица системы – это матрица, составленная только из коэффициентов при неизвестных, в данном примере матрица системы: . Расширенная матрица системы – это та же матрица системы плюс столбец свободных членов, в данном случае: . Любую из матриц можно для краткости называть просто матрицей.

После того, как расширенная матрица системы записана, с ней необходимо выполнить некоторые действия, которые также называются элементарными преобразованиями .

Существуют следующие элементарные преобразования:

1) Строки матрицы можно переставлять местами. Например, в рассматриваемой матрице можно безболезненно переставить первую и вторую строки:

2) Если в матрице есть (или появились) пропорциональные (как частный случай – одинаковые) строки, то следует удалить из матрицы все эти строки кроме одной. Рассмотрим, например матрицу . В данной матрице последние три строки пропорциональны, поэтому достаточно оставить только одну из них: .

3) Если в матрице в ходе преобразований появилась нулевая строка, то ее также следует удалить . Рисовать не буду, понятно, нулевая строка – это строка, в которой одни нули .

4) Строку матрицы можно умножить (разделить) на любое число, отличное от нуля . Рассмотрим, например, матрицу . Здесь целесообразно первую строку разделить на –3, а вторую строку – умножить на 2: . Данное действие очень полезно, поскольку упрощает дальнейшие преобразования матрицы.

5) Это преобразование вызывает наибольшие затруднения, но на самом деле ничего сложного тоже нет. К строке матрицы можно прибавить другую строку, умноженную на число , отличное от нуля. Рассмотрим нашу матрицу из практического примера: . Сначала я распишу преобразование очень подробно. Умножаем первую строку на –2: , и ко второй строке прибавляем первую строку умноженную на –2 : . Теперь первую строку можно разделить «обратно» на –2: . Как видите, строка, которую ПРИБАВЛЯЛИ не изменилась . Всегда меняется строка, К КОТОРОЙ ПРИБАВЛЯЮТ .

На практике так подробно, конечно, не расписывают, а пишут короче:

Еще раз: ко второй строке прибавили первую строку, умноженную на –2 . Умножают строку обычно устно или на черновике, при этом мысленный ход расчётов примерно такой:

«Переписываю матрицу и переписываю первую строку: »

«Сначала первый столбец. Внизу мне нужно получить ноль. Поэтому единицу вверху умножаю на –2: , и ко второй строке прибавляю первую: 2 + (–2) = 0. Записываю результат во вторую строку: »

«Теперь второй столбец. Вверху –1 умножаю на –2: . Ко второй строке прибавляю первую: 1 + 2 = 3. Записываю результат во вторую строку: »

«И третий столбец. Вверху –5 умножаю на –2: . Ко второй строке прибавляю первую: –7 + 10 = 3. Записываю результат во вторую строку: »

Пожалуйста, тщательно осмыслите этот пример и разберитесь в последовательном алгоритме вычислений, если вы это поняли, то метод Гаусса практически «в кармане». Но, конечно, над этим преобразованием мы еще поработаем.

Элементарные преобразования не меняют решение системы уравнений

! ВНИМАНИЕ : рассмотренные манипуляции нельзя использовать , если Вам предложено задание, где матрицы даны «сами по себе». Например, при «классических» действиях с матрицами что-то переставлять внутри матриц ни в коем случае нельзя!

Вернемся к нашей системе . Она практически разобрана по косточкам.

Запишем расширенную матрицу системы и с помощью элементарных преобразований приведем ее к ступенчатому виду :

(1) Ко второй строке прибавили первую строку, умноженную на –2. И снова: почему первую строку умножаем именно на –2? Для того чтобы внизу получить ноль, а значит, избавиться от одной переменной во второй строке.

(2) Делим вторую строку на 3.

Цель элементарных преобразований привести матрицу к ступенчатому виду: . В оформлении задания прямо так и отчеркивают простым карандашом «лестницу», а также обводят кружочками числа, которые располагаются на «ступеньках». Сам термин «ступенчатый вид» не вполне теоретический, в научной и учебной литературе он часто называется трапециевидный вид или треугольный вид .

В результате элементарных преобразований получена эквивалентная исходной система уравнений:

Теперь систему нужно «раскрутить» в обратном направлении – снизу вверх, этот процесс называется обратным ходом метода Гаусса .

В нижнем уравнении у нас уже готовый результат: .

Рассмотрим первое уравнение системы и подставим в него уже известное значение «игрек»:

Рассмотрим наиболее распространенную ситуацию, когда методом Гаусса требуется решить систему трёх линейных уравнений с тремя неизвестными.

Пример 1

Решить методом Гаусса систему уравнений:

Запишем расширенную матрицу системы:

Сейчас я сразу нарисую результат, к которому мы придём в ходе решения:

И повторюсь, наша цель – с помощью элементарных преобразований привести матрицу к ступенчатому виду. С чего начать действия?

Сначала смотрим на левое верхнее число:

Почти всегда здесь должна находиться единица . Вообще говоря, устроит и –1 (а иногда и другие числа), но как-то так традиционно сложилось, что туда обычно помещают единицу. Как организовать единицу? Смотрим на первый столбец – готовая единица у нас есть! Преобразование первое: меняем местами первую и третью строки:

Теперь первая строка у нас останется неизменной до конца решения . Уже легче.

Единица в левом верхнем углу организована. Теперь нужно получить нули вот на этих местах:

Нули получаем как раз с помощью «трудного» преобразования. Сначала разбираемся со второй строкой (2, –1, 3, 13). Что нужно сделать, чтобы на первой позиции получить ноль? Нужно ко второй строке прибавить первую строку, умноженную на –2 . Мысленно или на черновике умножаем первую строку на –2: (–2, –4, 2, –18). И последовательно проводим (опять же мысленно или на черновике) сложение, ко второй строке прибавляем первую строку, уже умноженную на –2 :

Результат записываем во вторую строку:

Аналогично разбираемся с третьей строкой (3, 2, –5, –1). Чтобы получить на первой позиции ноль, нужно к третьей строке прибавить первую строку, умноженную на –3 . Мысленно или на черновике умножаем первую строку на –3: (–3, –6, 3, –27). И к третьей строке прибавляем первую строку, умноженную на –3 :

Результат записываем в третью строку:

На практике эти действия обычно выполняются устно и записываются в один шаг:

Не нужно считать всё сразу и одновременно . Порядок вычислений и «вписывания» результатов последователен и обычно такой: сначала переписываем первую строку, и пыхтим себе потихонечку – ПОСЛЕДОВАТЕЛЬНО и ВНИМАТЕЛЬНО :


А мысленный ход самих расчётов я уже рассмотрел выше.

В данном примере это сделать легко, вторую строку делим на –5 (поскольку там все числа делятся на 5 без остатка). Заодно делим третью строку на –2, ведь чем меньше числа, тем проще решение:

На заключительном этапе элементарных преобразований нужно получить еще один ноль здесь:

Для этого к третьей строке прибавляем вторую строку, умноженную на –2 :


Попробуйте разобрать это действие самостоятельно – мысленно умножьте вторую строку на –2 и проведите сложение.

Последнее выполненное действие – причёска результата, делим третью строку на 3.

В результате элементарных преобразований получена эквивалентная исходной система линейных уравнений:

Круто.

Теперь в действие вступает обратный ход метода Гаусса. Уравнения «раскручиваются» снизу вверх.

В третьем уравнении у нас уже готовый результат:

Смотрим на второе уравнение: . Значение «зет» уже известно, таким образом:

И, наконец, первое уравнение: . «Игрек» и «зет» известны, дело за малым:

Ответ :

Как уже неоднократно отмечалось, для любой системы уравнений можно и нужно сделать проверку найденного решения, благо, это несложно и быстро.

Пример 2


Это пример для самостоятельного решения, образец чистового оформления и ответ в конце урока.

Следует отметить, что ваш ход решения может не совпасть с моим ходом решения, и это – особенность метода Гаусса . Но вот ответы обязательно должны получиться одинаковыми!

Пример 3

Решить систему линейных уравнений методом Гаусса

Запишем расширенную матрицу системы и с помощью элементарных преобразований приведем ее к ступенчатому виду:

Смотрим на левую верхнюю «ступеньку». Там у нас должна быть единица. Проблема состоит в том, что в первом столбце единиц нет вообще, поэтому перестановкой строк ничего не решить. В таких случаях единицу нужно организовать с помощью элементарного преобразования. Обычно это можно сделать несколькими способами. Я поступил так:
(1) К первой строке прибавляем вторую строку, умноженную на –1 . То есть, мысленно умножили вторую строку на –1 и выполнили сложение первой и второй строки, при этом вторая строка у нас не изменилась.

Теперь слева вверху «минус один», что нас вполне устроит. Кто хочет получить +1, может выполнить дополнительное телодвижение: умножить первую строку на –1 (сменить у неё знак).

(2) Ко второй строке прибавили первую строку, умноженную на 5. К третьей строке прибавили первую строку, умноженную на 3.

(3) Первую строку умножили на –1, в принципе, это для красоты. У третьей строки также сменили знак и переставили её на второе место, таким образом, на второй «ступеньке у нас появилась нужная единица.

(4) К третьей строке прибавили вторую строку, умноженную на 2.

(5) Третью строку разделили на 3.

Скверным признаком, который свидетельствует об ошибке в вычислениях (реже – об опечатке), является «плохая» нижняя строка. То есть, если бы у нас внизу получилось что-нибудь вроде , и, соответственно, , то с большой долей вероятности можно утверждать, что допущена ошибка в ходе элементарных преобразований.

Заряжаем обратный ход, в оформлении примеров часто не переписывают саму систему, а уравнения «берут прямо из приведенной матрицы». Обратный ход, напоминаю, работает, снизу вверх. Да тут подарок получился:

Ответ : .

Пример 4

Решить систему линейных уравнений методом Гаусса

Это пример для самостоятельного решения, он несколько сложнее. Ничего страшного, если кто-нибудь запутается. Полное решение и образец оформления в конце урока. Ваше решение может отличаться от моего решения.

В последней части рассмотрим некоторые особенности алгоритма Гаусса.
Первая особенность состоит в том, что иногда в уравнениях системы отсутствуют некоторые переменные, например:

Как правильно записать расширенную матрицу системы? Об этом моменте я уже рассказывал на уроке Правило Крамера. Матричный метод . В расширенной матрице системы на месте отсутствующих переменных ставим нули:

Кстати, это довольно легкий пример, поскольку в первом столбце уже есть один ноль, и предстоит выполнить меньше элементарных преобразований.

Вторая особенность состоит вот в чём. Во всех рассмотренных примерах на «ступеньки» мы помещали либо –1, либо +1. Могут ли там быть другие числа? В ряде случаев могут. Рассмотрим систему: .

Здесь на левой верхней «ступеньке» у нас двойка. Но замечаем тот факт, что все числа в первом столбце делятся на 2 без остатка – и другая двойка и шестерка. И двойка слева вверху нас устроит! На первом шаге нужно выполнить следующие преобразования: ко второй строке прибавить первую строку, умноженную на –1; к третьей строке прибавить первую строку, умноженную на –3. Таким образом, мы получим нужные нули в первом столбце.

Или еще такой условный пример: . Здесь тройка на второй «ступеньке» тоже нас устраивает, поскольку 12 (место, где нам нужно получить ноль) делится на 3 без остатка. Необходимо провести следующее преобразование: к третьей строке прибавить вторую строку, умноженную на –4, в результате чего и будет получен нужный нам ноль.

Метод Гаусса универсален, но есть одно своеобразие. Уверенно научиться решать системы другими методами (методом Крамера, матричным методом) можно буквально с первого раза – там очень жесткий алгоритм. Но вот чтобы уверенно себя чувствовать в методе Гаусса, следует «набить руку», и прорешать хотя бы 5-10 систем. Поэтому поначалу возможны путаница, ошибки в вычислениях, и в этом нет ничего необычного или трагического.

Дождливая осенняя погода за окном…. Поэтому для всех желающих более сложный пример для самостоятельного решения:

Пример 5

Решить методом Гаусса систему четырёх линейных уравнений с четырьмя неизвестными.

Такое задание на практике встречается не так уж и редко. Думаю, даже чайнику, который обстоятельно изучил эту страницу, интуитивно понятен алгоритм решения такой системы. Принципиально всё так же – просто действий больше.

Случаи, когда система не имеет решений (несовместна) или имеет бесконечно много решений, рассмотрены на уроке Несовместные системы и системы с общим решением . Там же можно закрепить рассмотренный алгоритм метода Гаусса.

Желаю успехов!

Решения и ответы:

Пример 2: Решение : Запишем расширенную матрицу системы и с помощью элементарных преобразований приведем ее к ступенчатому виду.


Выполненные элементарные преобразования:
(1) Ко второй строке прибавили первую строку, умноженную на –2. К третьей строке прибавили первую строку, умноженную на –1. Внимание! Здесь может возникнуть соблазн из третьей строки вычесть первую, крайне не рекомендую вычитать – сильно повышается риск ошибки. Только складываем!
(2) У второй строки сменили знак (умножили на –1). Вторую и третью строки поменяли местами. Обратите внимание , что на «ступеньках» нас устраивает не только единица, но еще и –1, что даже удобнее.
(3) К третьей строке прибавили вторую строку, умноженную на 5.
(4) У второй строки сменили знак (умножили на –1). Третью строку разделили на 14.

Обратный ход:

Ответ : .

Пример 4: Решение : Запишем расширенную матрицу системы и с помощью элементарных преобразований приведем ее к ступенчатому виду:

Выполненные преобразования:
(1) К первой строке прибавили вторую. Таким образом, организована нужная единица на левой верхней «ступеньке».
(2) Ко второй строке прибавили первую строку, умноженную на 7. К третьей строке прибавили первую строку, умноженную на 6.

Со второй «ступенькой» всё хуже , «кандидаты» на неё – числа 17 и 23, а нам нужна либо единичка, либо –1. Преобразования (3) и (4) будут направлены на получение нужной единицы

(3) К третьей строке прибавили вторую, умноженную на –1.
(4) Ко второй строке прибавили третью, умноженную на –3.
(3) К третьей строке прибавили вторую, умноженную на 4. К четвертой строке прибавили вторую, умноженную на –1.
(4) У второй строки сменили знак. Четвертую строку разделили на 3 и поместили вместо третьей строки.
(5) К четвертой строке прибавили третью строку, умноженную на –5.

Обратный ход:



Метод Гаусса прекрасно подходит для решения систем линейных алгебраических уравнений (СЛАУ). Он обладает рядом преимуществ по сравнению с другими методами:

  • во-первых, нет необходимости предварительно исследовать систему уравнений на совместность;
  • во-вторых, методом Гаусса можно решать не только СЛАУ, в которых число уравнений совпадает с количеством неизвестных переменных и основная матрица системы невырожденная, но и системы уравнений, в которых число уравнений не совпадает с количеством неизвестных переменных или определитель основной матрицы равен нулю;
  • в-третьих, метод Гаусса приводит к результату при сравнительно небольшом количестве вычислительных операций.

Краткий обзор статьи.

Сначала дадим необходимые определения и введем обозначения.

Далее опишем алгоритм метода Гаусса для простейшего случая, то есть, для систем линейных алгебраических уравнений, количество уравнений в которых совпадает с количеством неизвестных переменных и определитель основной матрицы системы не равен нулю. При решении таких систем уравнений наиболее отчетливо видна суть метода Гаусса, которая заключается в последовательном исключении неизвестных переменных. Поэтому метод Гаусса также называют методом последовательного исключения неизвестных. Покажем подробные решения нескольких примеров.

В заключении рассмотрим решение методом Гаусса систем линейных алгебраических уравнений, основная матрица которых либо прямоугольная, либо вырожденная. Решение таких систем имеет некоторые особенности, которые мы подробно разберем на примерах.

Навигация по странице.

Основные определения и обозначения.

Рассмотрим систему из p линейных уравнений с n неизвестными (p может быть равно n ):

Где – неизвестные переменные, – числа (действительные или комплексные), – свободные члены.

Если , то система линейных алгебраических уравнений называется однородной , в противном случае – неоднородной .

Совокупность значения неизвестных переменных , при которых все уравнения системы обращаются в тождества, называется решением СЛАУ .

Если существует хотя бы одно решение системы линейных алгебраических уравнений, то она называется совместной , в противном случае – несовместной .

Если СЛАУ имеет единственное решение, то она называется определенной . Если решений больше одного, то система называется неопределенной .

Говорят, что система записана в координатной форме , если она имеет вид
.

Эта система в матричной форме записи имеет вид , где – основная матрица СЛАУ, – матрица столбец неизвестных переменных, – матрица свободных членов.

Если к матрице А добавить в качестве (n+1)-ого столбца матрицу-столбец свободных членов, то получим так называемую расширенную матрицу системы линейных уравнений. Обычно расширенную матрицу обозначают буквой Т , а столбец свободных членов отделяют вертикальной линией от остальных столбцов, то есть,

Квадратная матрица А называется вырожденной , если ее определитель равен нулю. Если , то матрица А называется невырожденной .

Следует оговорить следующий момент.

Если с системой линейных алгебраических уравнений произвести следующие действия

  • поменять местами два уравнения,
  • умножить обе части какого-либо уравнения на произвольное и отличное от нуля действительное (или комплексное) число k ,
  • к обеим частям какого-либо уравнения прибавить соответствующие части другого уравнения, умноженные на произвольное число k ,

то получится эквивалентная система, которая имеет такие же решения (или также как и исходная не имеет решений).

Для расширенной матрицы системы линейных алгебраических уравнений эти действия будут означать проведение элементарных преобразований со строками:

  • перестановку двух строк местами,
  • умножение всех элементов какой-либо строки матрицы T на отличное от нуля число k ,
  • прибавление к элементам какой-либо строки матрицы соответствующих элементов другой строки, умноженных на произвольное число k .

Теперь можно переходить к описанию метода Гаусса.

Решение систем линейных алгебраических уравнений, в которых число уравнений равно числу неизвестных и основная матрица системы невырожденная, методом Гаусса.

Как бы мы поступили в школе, если бы получили задание найти решение системы уравнений .

Некоторые сделали бы так.

Заметим, что прибавив к левой части второго уравнения левую часть первого, а к правой части – правую, можно избавиться от неизвестных переменных x 2 и x 3 и сразу найти x 1 :

Подставляем найденное значение x 1 =1 в первое и третье уравнение системы:

Если умножить обе части третьего уравнения системы на -1 и прибавить их к соответствующим частям первого уравнения, то мы избавимся от неизвестной переменной x 3 и сможем найти x 2 :

Подставляем полученное значение x 2 =2 в третье уравнение и находим оставшуюся неизвестную переменную x 3 :

Другие поступили бы иначе.

Разрешим первое уравнение системы относительно неизвестной переменной x 1 и подставим полученное выражение во второе и третье уравнение системы, чтобы исключить из них эту переменную:

Теперь разрешим второе уравнение системы относительно x 2 и подставим полученный результат в третье уравнение, чтобы исключить из него неизвестную переменную x 2 :

Из третьего уравнения системы видно, что x 3 =3 . Из второго уравнения находим , а из первого уравнения получаем .

Знакомые способы решения, не правда ли?

Самое интересное здесь то, что второй способ решения по сути и есть метод последовательного исключения неизвестных, то есть, метод Гаусса. Когда мы выражали неизвестные переменные (сначала x 1 , на следующем этапе x 2 ) и подставляли их в остальные уравнения системы, мы тем самым исключали их. Исключение мы проводили до того момента, пока в последнем уравнении не осталась одна единственная неизвестная переменная. Процесс последовательного исключения неизвестных называется прямым ходом метода Гаусса . После завершения прямого хода у нас появляется возможность вычислить неизвестную переменную, находящуюся в последнем уравнении. С ее помощью из предпоследнего уравнения находим следующую неизвестную переменную и так далее. Процесс последовательного нахождения неизвестных переменных при движении от последнего уравнения к первому называется обратным ходом метода Гаусса .

Следует заметить, что когда мы выражаем x 1 через x 2 и x 3 в первом уравнении, а затем подставляем полученное выражение во второе и третье уравнения, то к такому же результату приводят следующие действия:

Действительно, такая процедура также позволяет исключить неизвестную переменную x 1 из второго и третьего уравнений системы:

Нюансы с исключением неизвестных переменных по методу Гаусса возникают тогда, когда уравнения системы не содержат некоторых переменных.

Например, в СЛАУ в первом уравнении отсутствует неизвестная переменная x 1 (иными словами, коэффициент перед ней равен нулю). Поэтому мы не можем разрешить первое уравнение системы относительно x 1 , чтобы исключить эту неизвестную переменную из остальных уравнений. Выходом из этой ситуации является перестановка местами уравнений системы. Так как мы рассматриваем системы линейных уравнений, определители основных матриц которых отличны от нуля, то всегда существует уравнение, в котором присутствует нужная нам переменная, и мы это уравнение можем переставить на нужную нам позицию. Для нашего примера достаточно поменять местами первое и второе уравнения системы , дальше можно разрешить первое уравнение относительно x 1 и исключить ее из остальных уравнений системы (хотя во втором уравнении x 1 уже отсутствует).

Надеемся, что суть Вы уловили.

Опишем алгоритм метода Гаусса.

Пусть нам требуется решить систему из n линейных алгебраических уравнений с n неизвестными переменными вида , и пусть определитель ее основной матрицы отличен от нуля.

Будем считать, что , так как мы всегда можем этого добиться перестановкой местами уравнений системы. Исключим неизвестную переменную x 1 из всех уравнений системы, начиная со второго. Для этого ко второму уравнению системы прибавим первое, умноженное на , к третьему уравнению прибавим первое, умноженное на , и так далее, к n-ому уравнению прибавим первое, умноженное на . Система уравнений после таких преобразований примет вид

где , а .

К такому же результату мы бы пришли, если бы выразили x 1 через другие неизвестные переменные в первом уравнении системы и полученное выражение подставили во все остальные уравнения. Таким образом, переменная x 1 исключена из всех уравнений, начиная со второго.

Далее действуем аналогично, но лишь с частью полученной системы, которая отмечена на рисунке

Для этого к третьему уравнению системы прибавим второе, умноженное на , к четвертому уравнению прибавим второе, умноженное на , и так далее, к n-ому уравнению прибавим второе, умноженное на . Система уравнений после таких преобразований примет вид

где , а . Таким образом, переменная x 2 исключена из всех уравнений, начиная с третьего.

Далее приступаем к исключению неизвестной x 3 , при этом действуем аналогично с отмеченной на рисунке частью системы

Так продолжаем прямой ход метода Гаусса пока система не примет вид

С этого момента начинаем обратный ход метода Гаусса: вычисляем x n из последнего уравнения как , с помощью полученного значения x n находим x n-1 из предпоследнего уравнения, и так далее, находим x 1 из первого уравнения.

Разберем алгоритм на примере.

Пример.

методом Гаусса.

Решение.

Коэффициент a 11 отличен от нуля, так что приступим к прямому ходу метода Гаусса, то есть, к исключению неизвестной переменной x 1 из всех уравнений системы, кроме первого. Для этого к левой и правой частям второго, третьего и четвертого уравнения прибавим левую и правую части первого уравнения, умноженные соответственно на , и :

Неизвестную переменную x 1 исключили, переходим к исключению x 2 . К левым и правым частям третьего и четвертого уравнений системы прибавляем левую и правую части второго уравнения, умноженные соответственно на и :

Для завершения прямого хода метода Гаусса нам осталось исключить неизвестную переменную x 3 из последнего уравнения системы. Прибавим к левой и правой частям четвертого уравнения соответственно левую и правую часть третьего уравнения, умноженную на :

Можно начинать обратный ход метода Гаусса.

Из последнего уравнения имеем ,
из третьего уравнения получаем ,
из второго ,
из первого .

Для проверки можно подставить полученные значения неизвестных переменных в исходную систему уравнений. Все уравнения обращаются в тождества, что говорит о том, что решение по методу Гаусса найдено верно.

Ответ:

А сейчас приведем решение этого же примера методом Гаусса в матричной форме записи.

Пример.

Найдите решение системы уравнений методом Гаусса.

Решение.

Расширенная матрица системы имеет вид . Сверху над каждым столбцом записаны неизвестные переменные, которым соответствуют элементы матрицы.

Прямой ход метода Гаусса здесь предполагает приведение расширенной матрицы системы к трапецеидальному виду с помощью элементарных преобразований. Этот процесс схож с исключением неизвестных переменных, которое мы проводили с системой в координатной форме. Сейчас Вы в этом убедитесь.

Преобразуем матрицу так, чтобы все элементы в первом столбце, начиная со второго, стали нулевыми. Для этого к элементам второй, третьей и четвертой строк прибавим соответствующие элементы первой строки умноженные на , и на соответственно:

Далее полученную матрицу преобразуем так, чтобы во втором столбце все элементы, начиная с третьего стали нулевыми. Это будет соответствовать исключению неизвестной переменной x 2 . Для этого к элементам третьей и четвертой строк прибавим соответствующие элементы первой строки матрицы, умноженные соответственно на и :

Осталось исключить неизвестную переменную x 3 из последнего уравнения системы. Для этого к элементам последней строки полученной матрицы прибавим соответствующие элементы предпоследней строки, умноженные на :

Следует отметить, что эта матрица соответствует системе линейных уравнений

которая была получена ранее после прямого хода.

Пришло время обратного хода. В матричной форме записи обратный ход метода Гаусса предполагает такое преобразование полученной матрицы, чтобы матрица, отмеченная на рисунке

стала диагональной, то есть, приняла вид

где – некоторые числа.

Эти преобразования аналогичны преобразованиям прямого хода метода Гаусса, но выполняются не от первой строки к последней, а от последней к первой.

Прибавим к элементам третьей, второй и первой строк соответствующие элементы последней строки, умноженные на , на и на соответственно:

Теперь прибавим к элементам второй и первой строк соответствующие элементы третьей строки, умноженные на и на соответственно:

На последнем шаге обратного хода метода Гаусса к элементам первой строки прибавляем соответствующие элементы второй строки, умноженные на :

Полученная матрица соответствует системе уравнений , откуда находим неизвестные переменные.

Ответ:

ОБРАТИТЕ ВНИМАНИЕ.

При использовании метода Гаусса для решения систем линейных алгебраических уравнений следует избегать приближенных вычислений, так как это может привести к абсолютно неверным результатам. Рекомендуем не округлять десятичные дроби. Лучше от десятичных дробей переходить к обыкновенным дробям.

Пример.

Решите систему из трех уравнений методом Гаусса .

Решение.

Отметим, что в этом примере неизвестные переменные имеют другое обозначение (не x 1 , x 2 , x 3 , а x, y, z ). Перейдем к обыкновенным дробям:

Исключим неизвестную x из второго и третьего уравнений системы:

В полученной системе во втором уравнении отсутствует неизвестная переменная y , а в третьем уравнении y присутствует, поэтому, переставим местами второе и третье уравнения:

На этом прямой ход метода Гаусса закончен (из третьего уравнения не нужно исключать y , так как этой неизвестной переменной уже нет).

Приступаем к обратному ходу.

Из последнего уравнения находим ,
из предпоследнего


из первого уравнения имеем

Ответ:

X = 10, y = 5, z = -20 .

Решение систем линейных алгебраических уравнений, в которых число уравнений не совпадает с числом неизвестных или основная матрица системы вырожденная, методом Гаусса.

Системы уравнений, основная матрица которых прямоугольная или квадратная вырожденная, могут не иметь решений, могут иметь единственное решение, а могут иметь бесконечное множество решений.

Сейчас мы разберемся, как метод Гаусса позволяет установить совместность или несовместность системы линейных уравнений, а в случае ее совместности определить все решения (или одно единственное решение).

В принципе процесс исключения неизвестных переменных в случае таких СЛАУ остается таким же. Однако следует подробно остановиться на некоторых ситуациях, которые могут возникнуть.

Переходим к самому важному этапу.

Итак, допустим, что система линейных алгебраических уравнений после завершения прямого хода метода Гаусса приняла вид и ни одно уравнение не свелось к (в этом случае мы бы сделали вывод о несовместности системы). Возникает логичный вопрос: «Что делать дальше»?

Выпишем неизвестные переменные, которые стоят на первом месте всех уравнений полученной системы:

В нашем примере это x 1 , x 4 и x 5 . В левых частях уравнений системы оставляем только те слагаемые, которые содержат выписанные неизвестные переменные x 1 , x 4 и x 5 , остальные слагаемые переносим в правую часть уравнений с противоположным знаком:

Придадим неизвестным переменным, которые находятся в правых частях уравнений, произвольные значения , где – произвольные числа:

После этого в правых частях всех уравнений нашей СЛАУ находятся числа и можно преступать к обратному ходу метода Гаусса.

Из последнего уравнений системы имеем , из предпоследнего уравнения находим , из первого уравнения получаем

Решением системы уравнений является совокупность значений неизвестных переменных

Придавая числам различные значения, мы будем получать различные решения системы уравнений. То есть, наша система уравнений имеет бесконечно много решений.

Ответ:

где – произвольные числа.

Для закрепления материала подробно разберем решения еще нескольких примеров.

Пример.

Решите однородную систему линейных алгебраических уравнений методом Гаусса.

Решение.

Исключим неизвестную переменную x из второго и третьего уравнений системы. Для этого к левой и правой части второго уравнения прибавим соответственно левую и правую части первого уравнения, умноженные на , а к левой и правой части третьего уравнения – левую и правую части первого уравнения, умноженные на :

Теперь исключим y из третьего уравнения полученной системы уравнений:

Полученная СЛАУ равносильна системе .

Оставляем в левой части уравнений системы только слагаемые, содержащие неизвестные переменные x и y , а слагаемые с неизвестной переменной z переносим в правую часть:

Решение систем линейных уравнений методом Гаусса. Пусть нам требуется найти решение системы из n линейных уравнений с n неизвестными переменными
определитель основной матрицы которой отличен от нуля.

Суть метода Гаусса состоит в последовательном исключении неизвестных переменных: сначала исключается x 1 из всех уравнений системы, начиная со второго, далее исключается x 2 из всех уравнений, начиная с третьего, и так далее, пока в последнем уравнении останется только неизвестная переменная x n . Такой процесс преобразования уравнений системы для последовательного исключения неизвестных переменных называется прямым ходом метода Гаусса . После завершения прямого хода метода Гаусса из последнего уравнения находитсяx n , с помощью этого значения из предпоследнего уравнения вычисляется x n-1 , и так далее, из первого уравнения находится x 1 . Процесс вычисления неизвестных переменных при движении от последнего уравнения системы к первому называется обратным ходом метода Гаусса .

Кратко опишем алгоритм исключения неизвестных переменных.

Будем считать, что , так как мы всегда можем этого добиться перестановкой местами уравнений системы. Исключим неизвестную переменную x 1 из всех уравнений системы, начиная со второго. Для этого ко второму уравнению системы прибавим первое, умноженное на , к третьему уравнению прибавим первое, умноженное на , и так далее, к n-ому уравнению прибавим первое, умноженное на . Система уравнений после таких преобразований примет вид

где , а .

К такому же результату мы бы пришли, если бы выразили x 1 через другие неизвестные переменные в первом уравнении системы и полученное выражение подставили во все остальные уравнения. Таким образом, переменная x 1 исключена из всех уравнений, начиная со второго.

Далее действуем аналогично, но лишь с частью полученной системы, которая отмечена на рисунке

Для этого к третьему уравнению системы прибавим второе, умноженное на , к четвертому уравнению прибавим второе, умноженное на , и так далее, к n-ому уравнению прибавим второе, умноженное на . Система уравнений после таких преобразований примет вид

где , а . Таким образом, переменная x 2 исключена из всех уравнений, начиная с третьего.

Далее приступаем к исключению неизвестной x 3 , при этом действуем аналогично с отмеченной на рисунке частью системы

Так продолжаем прямой ход метода Гаусса пока система не примет вид

С этого момента начинаем обратный ход метода Гаусса: вычисляем x n из последнего уравнения как , с помощью полученного значения x n находим x n-1 из предпоследнего уравнения, и так далее, находим x 1 из первого уравнения.

Пример.

Решите систему линейных уравнений методом Гаусса.

Пусть дана система , ∆≠0. (1)
Метод Гаусса – это метод последовательного исключения неизвестных.

Суть метода Гаусса состоит в преобразовании (1) к системе с треугольной матрицей , из которой затем последовательно (обратным ходом) получаются значения всех неизвестных. Рассмотрим одну из вычислительных схем. Эта схема называется схемой единственного деления. Итак, рассмотрим эту схему. Пусть a 11 ≠0 (ведущий элемент) разделим на a 11 первое уравнение. Получим
(2)
Пользуясь уравнением (2), легко исключить неизвестные x 1 из остальных уравнений системы (для этого достаточно из каждого уравнения вычесть уравнение (2) предварительно умноженное на соответствующий коэффициент при x 1), то есть на первом шаге получим
.
Иными словами, на 1 шаге каждый элемент последующих строк, начиная со второй, равен разности между исходным элементом и произведением его «проекции» на первый столбец и первую (преобразованную) строку.
Вслед за этим оставив первое уравнение в покое, над остальными уравнениями системы, полученной на первом шаге, совершим аналогичное преобразование: выберем из их числа уравнение с ведущим элементом и исключим с его помощью из остальных уравнений x 2 (шаг 2).
После n шагов вместо (1) получим равносильную систему
(3)
Таким образом, на первом этапе мы получим треугольную систему (3). Этот этап называется прямым ходом.
На втором этапе (обратный ход) мы находим последовательно из (3) значения x n , x n -1 , …, x 1 .
Обозначим полученное решение за x 0 . Тогда разность ε=b-A·x 0 называется невязкой .
Если ε=0, то найденное решение x 0 является верным.

Вычисления по методу Гаусса выполняются в два этапа:

  1. Первый этап называется прямым ходом метода. На первом этапе исходную систему преобразуют к треугольному виду.
  2. Второй этап называется обратным ходом. На втором этапе решают треугольную систему, эквивалентную исходной.
Коэффициенты а 11 , а 22 , …, называют ведущими элементами.
На каждом шаге предполагалось, что ведущий элемент отличен от нуля. Если это не так, то в качестве ведущего можно использовать любой другой элемент, как бы переставив уравнения системы.

Назначение метода Гаусса

Метод Гаусса предназначен для решения систем линейных уравнений. Относится к прямым методам решения.

Виды метода Гаусса

  1. Классический метод Гаусса;
  2. Модификации метода Гаусса. Одной из модификаций метода Гаусса является схема с выбором главного элемента. Особенностью метода Гаусса с выбором главного элемента является такая перестановка уравнений, чтобы на k -ом шаге ведущим элементом оказывался наибольший по модулю элемент k -го столбца.
  3. Метод Жордано-Гаусса;
Отличие метода Жордано-Гаусса от классического метода Гаусса состоит в применении правила прямоугольника , когда направление поиска решения происходит по главной диагонали (преобразование к единичной матрице). В методе Гаусса направление поиска решения происходит по столбцам (преобразование к системе с треугольной матрицей).
Проиллюстрируем отличие метода Жордано-Гаусса от метода Гаусса на примерах.

Пример решения методом Гаусса
Решим систему:

Для удобства вычислений поменяем строки местами:

Умножим 2-ую строку на (2). Добавим 3-ую строку к 2-ой

Умножим 2-ую строку на (-1). Добавим 2-ую строку к 1-ой

Из 1-ой строки выражаем x 3:
Из 2-ой строки выражаем x 2:
Из 3-ой строки выражаем x 1:

Пример решения методом Жордано-Гаусса
Эту же СЛАУ решим методом Жордано-Гаусса.

Последовательно будем выбирать разрешающий элемент РЭ, который лежит на главной диагонали матрицы.
Разрешающий элемент равен (1).

НЭ = СЭ – (А*В)/РЭ
РЭ – разрешающий элемент (1), А и В – элементы матрицы, образующие прямоугольник с элементами СТЭ и РЭ.
Представим расчет каждого элемента в виде таблицы:

x 1x 2x 3B
1 / 1 = 12 / 1 = 2-2 / 1 = -21 / 1 = 1


Разрешающий элемент равен (3).
На месте разрешающего элемента получаем 1, а в самом столбце записываем нули.
Все остальные элементы матрицы, включая элементы столбца B, определяются по правилу прямоугольника.
Для этого выбираем четыре числа, которые расположены в вершинах прямоугольника и всегда включают разрешающий элемент РЭ.
x 1x 2x 3B
0 / 3 = 03 / 3 = 11 / 3 = 0.334 / 3 = 1.33


Разрешающий элемент равен (-4).
На месте разрешающего элемента получаем 1, а в самом столбце записываем нули.
Все остальные элементы матрицы, включая элементы столбца B, определяются по правилу прямоугольника.
Для этого выбираем четыре числа, которые расположены в вершинах прямоугольника и всегда включают разрешающий элемент РЭ.
Представим расчет каждого элемента в виде таблицы:

Ответ : x 1 = 1, x 2 = 1, x 3 = 1

Реализация метода Гаусса

Метод Гаусса реализован на многих языках программирования, в частности: Pascal, C++, php, Delphi , а также имеется реализация метода Гаусса в онлайн режиме .

Использование метода Гаусса

Применение метода Гаусса в теории игр

В теории игр при отыскании максиминной оптимальной стратегии игрока составляется система уравнений, которая решается методом Гаусса.

Применение метода Гаусса при решении дифференциальных уравнений

Для поиска частного решения дифференциального уравнения сначала находят производные соответствующей степени для записанного частного решения (y=f(A,B,C,D)), которые подставляют в исходное уравнение. Далее, чтобы найти переменные A,B,C,D составляется система уравнений, которая решается методом Гаусса.

Применение метода Жордано-Гаусса в линейном программировании

В линейном программировании, в частности в симплекс-методе для преобразования симплексной таблицы на каждой итерации используется правило прямоугольника, в котором используется метод Жордано-Гаусса.

Одним из универсальных и эффективных методов реше­ния линейных алгебраических систем является метод Гаусса , состо­ящий в последовательном исключении неизвестных.

Напомним, две системы называются эквивалентными (равносильными), если множества их решений совпадают. Другими словами, системы эквивалентны, если каждое решение одной из них является решением другой и наоборот. Эквивалентные системы получаются приэлементарных преобразованиях уравнений системы:

    умножение обеих частей уравнения на число отличное от нуля;

    прибавление к некоторому уравнению соответствующих частей другого уравнения, умноженных на число отличное от нуля;

    перестановка двух уравнений.

Пусть дана система уравнений

Процесс решения этой системы по методу Гаусса состоит из двух этапов. На первом этапе (прямой ход) система с помощью элементарных преобразований приводится к ступен­чатому , илитреугольному виду, а на втором этапе (обратный ход) идет последовательное, начиная с последнего по номеру переменного, определение неизвестных из полученной ступенчатой системы.

Предположим, что коэффициент данной системы
, в против­ном случае в системе первую строку можно поменять местами с любой другой строкой так, чтобы коэффициент прибыл отличен от нуля.

Преобразуем систему, исключив неизвестное во всех уравне­ниях, кроме первого. Для этого умножим обе части первого уравнения наи сложим почленно со вторым уравнением системы. Затем умножим обе части первого уравнения наи сложим с третьим уравнением системы. Продолжая этот процесс, получим эквивалент­ную систему

Здесь
– новые значения коэффициентов и свободных членов, которые получаются после первого шага.

Аналогичным образом, считая главным элементом
, исклю­чим неизвестноеиз всех уравнений системы, кроме первого и второго. Продолжим этот процесс, пока это возможно, в результате получим ступенчатую систему

,

где ,
,…,– главные элементы системы
.

Если в процессе приведения системы к ступенчатому виду появятся уравнения , т. е. равенства вида
, их отбрасывают, так как им удовлетворяют любые наборы чисел
. Если же при
появится уравнение вида, которое не имеет решений, то это свидетельствует о несовместности системы.

При обратном ходе из последнего уравнения преобразованной сту­пенчатой системы выражается первое неизвестное через все остальные неизвестные
, которые называютсвободными . Затем выражение переменнойиз последнего уравнения системы подставляется в предпоследнее уравнение и из него выражается переменная
. Аналогичным образом последовательно определяются переменные
. Переменные
, выраженные через свободные переменные, называютсябазисными (зависимыми). В результате получается общее решение системы линейных уравнений.

Чтобы найти частное решение системы, свободным неизвестным
в общем решении придаются произвольные значения и вычисляются значения переменных
.

Технически удобнее подвергать элементарным преобразованиям не сами уравнения системы, а расширенную матрицу системы

.

Метод Гаусса – универсальный метод, который позволяет решать не только квадратные, но и прямоугольные системы, в которых число неизвестных
не равно числу уравнений
.

Достоинство этого метода состоит также в том, что в процессе решения мы одновременно исследуем систему на совместность, так как, приведя расширенную матрицу
к ступенчатому виду, легко определить ранги матрицыи расширенной матрицы
и применитьтеорему Кронекера – Капелли .

Пример 2.1 Методом Гаусса решить систему

Решение . Число уравнений
и число неизвестных
.

Составим расширенную матрицу системы, приписав справа от матрицы коэффициентов столбец свободных членов.

Приведём матрицу к треугольному виду; для этого будем получать «0» ниже элементов, стоящих на главной диагонали с помощью элементарных преобразований.

Чтобы получить «0» во второй позиции первого столбца, умножим первую строку на (-1) и прибавим ко второй строке.

Это преобразование запишем числом (-1) против первой строки и обозначим стрелкой, идущей от первой строки ко второй строке.

Для получения «0» в третьей позиции первого столбца, умножим первую строку на (-3) и прибавим к третьей строке; покажем это действие с помощью стрелки, идущей от первой строки к третьей.




.

В полученной матрице, записанной второй в цепочке матриц, получим «0» во втором столбце в третьей позиции. Для этого умножили вторую строку на (-4) и прибавили к третьей. В полученной матрице вторую строку умножим на (-1), а третью – разделим на (-8). Все элементы этой матрицы, лежащие ниже диагональных элементов – нули.

Так как , система является совместной и определенной.

Соответствующая последней матрице система уравнений имеет треугольный вид:

Из последнего (третьего) уравнения
. Подставим во второе уравнение и получим
.

Подставим
и
в первое уравнение, найдём


.

С++ программная реализация метода Гаусса

Вычислительная схема метода Гаусса состоит из двух этапов. Первый этап заключается в приведении системы к трапециевидной. Этот этап называется прямым ходом. Второй этап – определение неизвестных – называется обратным ходом.

Прямой ход метода Гаусса состоит в последовательном исключении коэффициентов при неизвестных начиная с первого столбца.

Прямой ход реализуется по следующим формулам (индекс k в круглых скобках означает номер цикла – номер столбца).

Умножение k-й строки на число

.  (1)

Вычитание k-й строки из j-й строки

.  (2)

.  (3)

Обратный ход – вычисление неизвестных – реализуется по следующим формулам, начиная с последнего уравнения системы

.  (4)

Код C++

#include <iostream>

using namespace std;

int n, i, j, k;

double d, s;

int main()

{

cout “Poryadok: ”

cin >> n;

double **a = new double *[n];

for (i = 0; i

a[i] = new double [n];

double **a1 = new double *[n];

for (i = 0; i

a1[i] = new double [n];

double *b = new double [n];

double *x = new double [n];

cout “Vvedite koefficienty i svobodnye chleny ”

for (i = 1; i

{

for (j = 1; j

{

cout “a[ ” “,” “]= “;

cin >> a[i][j];

a1[i][j] = a[i][j];

}

cout “b,[ ” “]= “;

cin >> b[i];

}

for (k = 1; k // прямой ход

{

for (j = k + 1; j

{

d = a[j][k] / a[k][k]; // формула (1)

for (i = k; i

{

a[j][i] = a[j][i] – d * a[k][i]; // формула (2)

}

b[j] = b[j] – d * b[k]; // формула (3)

}

}

for (k = n; k >= 1; k–) // обратный ход

{

d = 0;

for (j = k + 1; j

{

s = a[k][j] * x[j]; // формула (4)

d = d + s; // формула (4)

}

x[k] = (b[k] – d) / a[k][k]; // формула (4)

}

cout “Korni sistemy: ”

for( i = 1; i

cout “x[” “]=” ” ”

return 0;

}

Метод Гаусса – определение с примерами решения

Содержание:

  1. Опишем метод Гаусса подробнее
  2. Примеры с решением

Одним из наиболее универсальных и эффективных методов решений линейных алгебраических систем является метод Гаусса, состоящий в последовательном исключении неизвестных.

Пусть дана система уравнений (4.3)

Процесс решения по методу Гаусса состоит из двух этапов. На нервом этапе (прямой ход) система приводится к i ступенчатому (в частности, треугольному) виду.

Приведенная ниже система имеет ступенчатый вид

где

Коэффициенты называются главными элементами системы. На втором этапе (обратный ход) идет последовательное определение неизвестных из этой ступенчатой системы.

По этой ссылке вы найдёте полный курс лекций по высшей математике:

Опишем метод Гаусса подробнее

Прямой ход. Будем считать, что элемент (если то первым в системе запишем уравнение, в котором коэффициент при отличен от нуля). Преобразуем систему (4.3), исключив неизвестное во всех уравнениях, кроме первого (используя элементарные преобразования системы).

Для этого умножим обе части первого уравнения на и сложим почленно со вторым уравнением системы.

Затем умножим обе части первого уравнения на и сложим с третьим уравнением сиап стемы. Продолжая этот процесс, получим эквивалентную систему

Здесь

новые значения коэффициентов и правых частей, которые полу чаю юя после первого шага. Аналогичным образом, считая главным элементом исключим неизвестное из всех уравнений системы, кроме первого и второго, и так далее.

Продолжаем этот процесс, пока это возможно. Если в процессе приведения системы (4.3) к ступенчатому виду появятся нулевые уравнения, т. е. равенства вида их отбрасывают. Если же появится уравнение вида то это свидетельствует о несовместности системы. Второй этап {обратный ход) заключается в решении ступенчатой системы.

Ступенчатая система уравнений, вообще говоря, имеет бесчисленное множество решений. В последнем уравнении этой системы выражаем первое неизвестное через остальные неизвестные Затем подставляем значение в предпоследнее уравнение системы и выражаем затем находим Придавая свободным неизвестным произвольные значения, получим бесчисленное множество решений системы.

Возможно вам будут полезны данные страницы:

Замечания: 1. Если ступенчатая система оказывается треугольной, т. е. то исходная система имеет единственное решение. Из последнего уравнения находим хп, из предпоследнего уравнения далее поднимаясь по системе вверх, найдем все остальные неизвестные На практике удобнее работать не с системой (4.3), а с расширенной ее матрицей, выполняя все элементарные преобразования над ее строками. Удобно, чтобы коэффициент был равен 1 (уравнения переставить местами, либо разделить обе части уравнения на ).

Примеры с решением

Пример 4.4.

Решить систему методом Гаусса:

Решение:

В результате элементарных преобразований над расширенной матрицей системы

исходная система свелась к ступенчатой:

Поэтому общее решение системы: Если положить, например, то найдем одно из частных решений этой системы

Пример 4.5.

Решить систему методом Гаусса:

Решение:

Произведем элементарные преобразования над строчками расширенной матрицы системы:

Полученная матрица соответствует системе

Осуществляя обратный ход, находим Рассмотрим решение системы (1.25) т линейных уравнений с п неизвестными. Заметим, что матрица коэффициентов системы не обязательно должна быть квадратной.

Предлагаемые методы решения систем линейных алгебраических уравнений сводятся к элементарным преобразованиям над уравнениями системы.

Преобразования Гаусса удобно проводить, осуществляя преобразования не самих уравнений, а строк матрицы их коэффициентов.

Рассмотрим матрицу

(1.33) называемую расширенной матрицей системы (1.25), так как в нее, кроме коэффициентов матрицы А системы (1.25), дополнительно включен столбец свободных членов В. Метод Гаусса (метод последовательного исключения неизвестных. Рассмотрим систему (1.25) в случае

Суть метода Гаусса заключается в том, что с помощью элементарных преобразований расширенная матрица системы (1.33) приводится к равносильной матрице ступенчатого вида по алгоритму поиска ранга матрицы (см. пример 1.13). Это и есть прямой ход метода Гаусса.

На основании полученной ступенчатой матрицы составляется новая система уравнений, равносильная исходной, из которой последовательно, начиная с последнего уравнения, находятся все неизвестные; это суть обратного хода метода Гаусса.

Пример 1.18.

Решить систему уравнений методом Гаусса:

Решение:

Расширенная матрица системы имеет вид

Используя элементарные преобразования (см. пример 1.14), сведем эту матрицу к ступенчатой, не забывая при этом, что теперь в строке уже не три, а четыре элемента.

Вспомнив, что до черты стоят коэффициенты системы уравнений, а после нее — столбец свободных членов, выпишем получившуюся систему уравнений:

Теперь применим обратный ход метода Гаусса. Из последнего 4 2 тл уравнения полученной системы найдем

Из второго уравнения найдем Аналогично найдем из первого уравнения, подставив в него уже наиденные два неизвестных

Получим решение системы

Предлагаем читателю убедиться, что найденные числа образуют решение данной системы. Расширенная матрица системы. Ступенчатая матрица. Метод Гаусса. Коэффициенты системы (1.1) удобно объединить в прямоугольную таблицу, называемую матрицей системы. Для матрицы принято обозначение:

Матрица содержит т горизонтальных рядов, называемых строками, и вертикальных рядов, называемых столбцами, числа называются ее элементами. Таким образом, первый индекс элемента — это номер строки (номер уравнения системы (1.1)), а второй индекс — номер столбца (или номер неизвестного коэффициентом при котором является уравнении системы (1.1)).

Например, матрица

квадратная матрица 3-го — единичная матрица 2-го порядка. Если к матрице А добавить столбец из свободных членов, то получим так называемую расширенную матрицу А* системы, содержащую всю информацию о системе:

Для системы из примера 1.1 матрицей системы является а расширенной матрицей этой системы является матрица На практике элементарным преобразованиям подвергают не саму систему, а ее расширенную матрицу.

Преобразованиям двух типов над системой (1.1) соответствуют два типа элементарных преобразований над строками матрицы 1-й тип — перестановка местами двух любых ее строк; 2-й тип — сложение соответствующих элементов двух любых строк, все элементы одной из которых предварительно умножены на одно и то же число.

Целью элементарных преобразований является приведение расширенной матрицы системы (1.1) к так называемой ступенчатой форме.

Определение 1.6. Матрица называется ступенчатой, если для нее выполняются следующие условия: 1) если какая-либо строка данной матрицы состоит из нулей, то и все последующие строки также состоят из нулей; 2) если — первый ненулевой элемент строки, а — первый ненулевой элемент строки,то Так, например, матрица

является ступенчатой.

Матрица из одной строки считается ступенчатой по определению. Теорема 1.2. Любую матрицу Л конечным числом элементарных преобразований первого и второго типов можно преобразовать в ступенчатую матрицу.

Пример 1.7.

Привести к ступенчатому виду матрицу

Решение:

Выполним следующие элементарные преобразования над матрицей

1) к элементам второй строки прибавим элементы первой строки и из элементов третьей строки вычтем элементы первой строки, в результате преобразуется к виду: — расширенная матрица системы.

2) переставим вторую и третью строки:

3) из третьей строки полученной матрицы вычтем вторую строку, умноженную на 3, получим:

На приведении расширенной матрицы системы (1.1) к ступенчатой матрице основан метод Гаусса, или метод последовательного исключения неизвестных. Система линейных уравнений с расширенной ступенчатой матрицей называется ступенчатой системой, по теореме 1.1 она будет равносильна соответствующей системе в форме (1.1). Приведение системы (1.1) к ступенчатой форме называется прямым ходом метода Гаусса.

Решение полученной ступенчатой системы называется обратным ходом метода Гаусса. Он может быть выполнен как в форме последовательного определения неизвестных, начиная с последнего уравнения ступенчатой системы, так и в форме преобразования матрицы к ступенчатой матрице специального вида.

Пример 1.8.

Решить методом Гаусса систему уравнений

Решение:

Прямой ход метода Гаусса. В примере 1.7 матрица при помощи элементарных преобразований приведена к ступенчатой матрице

Теперь матрице сопоставим систему, для которой она будет расширенной матрицей:

Обратный ход метода Гаусса. 1-й способ. Имеем: . 2-й способ.

Умножим последнюю строку матрицы на 1 /5, сложим со второй строкой, после чего к первой строке прибавим последнюю, умноженную на (-2), с целью получить нули в третьем столбце:

Напишем систему с расширенной матрицей

Ответ: система совместная и определенная, она имеет единственное решение:

Метод Гаусса

Определение и описание метода Гаусса

Метод преобразований Гаусса (также известный как преобразование методом последовательного исключения неизвестных переменных из уравнения или матрицы) для решения систем линейных уравнений представляет собой классический методом решения системы алгебраических уравнений (СЛАУ). Также этот классический метод используют для решения таких задач как получение обратных матриц и определения ранговости матрицы.

Преобразование с помощью метода Гаусса заключается в совершении небольших (элементарных) последовательных изменениях системы линейных алгебраических уравнений, приводящих к исключению переменных из неё сверху вниз с образованием новой треугольной системы уравнений, являющейся равносильной исходной.

Определение 1

Эта часть решения носит название прямого хода решения Гаусса, так как весь процесс осуществляется сверху вниз.

После приведения исходной системы уравнений к треугольной осуществляется нахождение всех переменных системы снизу вверх (то есть первые найденные переменные занимают находятся именно на последних строчках системы или матрицы). Эта часть решения известна также как обратный ход решения методом Гаусса. Заключается его алгоритм в следующем: сначала вычисляется переменные, находящиеся ближе всего к низу системы уравнений или матрицы, затем полученные значения подставляются выше и таким образом находится ещё одна переменная и так далее.

Описание алгоритма метода Гаусса

Последовательность действий для общего решения системы уравнения методом Гаусса заключается в поочередном применении прямого и обратного хода к матрице на основе СЛАУ. Пусть исходная система уравнений имеет следующий вид:

$\begin{cases} a_{11} \cdot x_1 +…+ a_{1n} \cdot x_n = b_1 \\ … \\ a_{m1} \cdot x_1 + a_{mn} \cdot x_n = b_m \end{cases}$

Чтобы решить СЛАУ методом Гаусса, необходимо записать исходную систему уравнений в виде матрицы:

Готовые работы на аналогичную тему

$A = \begin{pmatrix} a_{11} & … & a_{1n} \\ \vdots & … & \vdots \\ a_{m1} & … & a_{mn} \end{pmatrix}$, $b=\begin{pmatrix} b_1 \\ \vdots \\ b_m \end{pmatrix}$

Матрица $A$ называется основной матрицей и представляет собой записанные по порядку коэффициенты при переменных, а $b$ называется столбцом её свободных членов. Матрица $A$, записанная через черту со столбцом свободных членов называется расширенной матрицей:

$A = \begin{array}{ccc|c} a_{11} & … & a_{1n} & b_1 \\ \vdots & … & \vdots & …\\ a_{m1} & … & a_{mn} & b_m \end{array}$

Теперь необходимо с помощью элементарных преобразований над системой уравнений (или над матрицей, так как это удобнее) привести её к следующему виду:

$\begin{cases} α_{1j_{1}} \cdot x_{j_{1}} + α_{1j_{2}} \cdot x_{j_{2}}…+ α_{1j_{r}} \cdot x_{j_{r}} +… α_{1j_{n}} \cdot x_{j_{n}} = β_1 \\ α_{2j_{2}} \cdot x_{j_{2}}…+ α_{2j_{r}} \cdot x_{j_{r}} +… α_{2j_{n}} \cdot x_{j_{n}} = β_2 \\ …\\ α_{rj_{r}} \cdot x_{j_{r}} +… α_{rj_{n}} \cdot x_{j_{n}} = β_r \\ 0 = β_(r+1) \\ … \\ 0 = β_m \end{cases}$ (1)

Матрица, полученная из коэффициентов преобразованной системы уравнения (1) называется ступенчатой, вот так обычно выглядят ступенчатые матрицы:

$A = \begin{array}{ccc|c} a_{11} & a_{12} & a_{13} & b_1 \\ 0 & a_{22} & a_{23} & b_2\\ 0 & 0 & a_{33} & b_3 \end{array}$

Для этих матриц характерен следующий набор свойств:

  1. Все её нулевые строки стоят после ненулевых
  2. Если некоторая строка матрицы с номером $k$ ненулевая, то в предыдущей строчке этой же матрицы нулей меньше, чем в этой, обладающей номером $k$.

После получения ступенчатой матрицы необходимо подставить полученные переменные в оставшиеся уравнения (начиная с конца) и получить оставшиеся значения переменных.

Основные правила и разрешаемые преобразования при использовании метода Гаусса

При упрощении матрицы или системы уравнений этим методом нужно использовать только элементарные преобразования.

Таким преобразованиями считаются операции, которые возможно применять к матрице или системе уравнений без изменения её смысла:

  • перестановка нескольких строк местами,
  • прибавление или вычитание из одной строчки матрицы другой строчки из неё же,
  • умножение или деление строчки на константу, не равную нулю,
  • строчку, состоящую из одних нулей, полученную в процессе вычисления и упрощения системы, нужно удалить,
  • Также нужно удалить лишние пропорциональные строчки, выбрав для системы единственную из них с более подходящими и удобными для дальнейших вычислений коэффициентами.

Все элементарные преобразования являются обратимыми.

Разбор трёх основных случаев, возникающих при решении линейных уравнений используя метод простых преобразований Гаусса

Различают три возникающих случая при использовании метода Гаусса для решения систем:

  1. Когда система несовместная, то есть у неё нет каких-либо решений
  2. У системы уравнений есть решение, причём единственное, а количество ненулевых строк и столбцов в матрице равно между собой.
  3. Система имеет некое количество или множество возможных решений, а количество строк в ней меньше чем количество столбцов.

Исход решения с несовместной системой

Для этого варианта при решении матричного уравнения методом Гаусса характерно получение какой-то строчки с невозможностью выполнения равенства. Поэтому при возникновении хотя бы одного неправильного равенства полученная и исходная системы не имеют решений вне зависимости от остальных уравнений, которые они содержат. Пример несовместной матрицы:

$\begin{array}{ccc|c} 2 & -1 & 3 & 0 \\ 1 & 0 & 2 & 0\\ 0 & 0 & 0 & 1 \end{array}$

В последней строчке возникло невыполняемое равенство: $0 \cdot x_{31} + 0 \cdot x_{32} + 0 \cdot x_{33} = 1$.

Система уравнений, у которой есть только одно решение

Данные системы после приведения к ступенчатой матрице и удаления строчек с нулями имеют одинаковое количество строк и столбцов в основной матрице. Вот простейший пример такой системы:

$\begin{cases} x_1 – x_2 = -5 \\ 2 \cdot x_1 + x_2 = -7 \end{cases}$

Запишем её в виде матрицы:

$\begin{array}{cc|c} 1 & -1 & -5 \\ 2 & 1 & -7 \end{array}$

Чтобы привести первую ячейку второй строчки к нулю, домножим верхнюю строку на $-2$ и вычтем её из нижней строчки матрицы, а верхнюю строчку оставим в исходном виде, в итоге имеем следующее:

$\begin{array}{cc|c} 1 & -1 & -5 \\ 0 & 3 & 10 \end{array}$

Этот пример можно записать в виде системы:

$\begin{cases} x_1 – x_2 = -5 \\ 3 \cdot x_2 = 10 \end{cases}$

Из нижнего уравнения выходит следующее значение $x$: $x_2 = 3 \frac{1}{3}$. Подставим это значение в верхнее уравнение: $x_1 – 3 \frac{1}{3}$, получаем $x_1 = 1 \frac{2}{3}$.

Система, обладающая множеством возможных вариантов решений

Для этой системы характерно меньшее количество значащих строк, чем количество столбцов в ней (учитываются строки основной матрицы).

Переменные в такой системе делятся на два вида: базисные и свободные. При преобразовании такой системы содержащиеся в ней основные переменные необходимо оставить в левой области до знака “=”, а остальные переменные перенести в правую часть равенства.

У такой системы есть только некое общее решение.

Разберём следующую систему уравнений:

$\begin{cases} 2y_1 + 3y_2 + x_4 = 1 \\ 5y_3 – 4y_4 = 1 \end{cases}$

Запишем её в виде матрицы:

$\begin{array}{cccc|c} 2 & 3 & 0 & 1 & 1 \\ 0 & 0 & 5 & 4 & 1 \\ \end{array}$

Наша задача найти общее решение системы. Для этой матрицы базисными переменными будут $y_1$ и $y_3$ (для $y_1$ – так как он стоит на первом месте, а в случае $y_3$ – располагается после нулей).

В качестве базисных переменных выбираем именно те, которые первые в строке не равны нулю.

Оставшиеся переменные называются свободными, через них нам необходимо выразить базисные.

Используя так называемый обратный ход, разбираем систему снизу вверх, для этого сначала выражаем $y_3$ из нижней строчки системы:

$5y_3 – 4y_4 = 1$

$5y_3 = 4y_4 + 1$

$y_3 = \frac{4/5}y_4 + \frac{1}{5}$.

Теперь в верхнее уравнение системы $2y_1 + 3y_2 + y_4 = 1$ подставляем выраженное $y_3$: $2y_1 + 3y_2 – (\frac{4}{5}y_4 + \frac{1}{5}) + y_4 = 1$

Выражаем $y_1$ через свободные переменные $y_2$ и $y_4$:

$2y_1 + 3y_2 – \frac{4}{5}y_4 – \frac{1}{5} + y_4 = 1$

$2y_1 = 1 – 3y_2 + \frac{4}{5}y_4 + \frac{1}{5} – y_4$

$2y_1 = -3y_2 – \frac{1}{5}y_4 + \frac{6}{5}$

$y_1 = -1.5x_2 – 0.1y_4 + 0.6$

Решение готово.

Пример 1

Решить слау методом Гаусса. Примеры. Пример решения системы линейных уравнений заданных матрицей 3 на 3 используя метод Гаусса

$\begin{cases} 4x_1 + 2x_2 – x_3 = 1 \\ 5x_1 + 3x_2 – 2x^3 = 2\\ 3x_1 + 2x_2 – 3x_3 = 0 \end{cases}$

Запишем нашу систему в виде расширенной матрицы:

$\begin{array}{ccc|c} 4 & 2 & -1 & 1 \\ 5 & 3 & -2 & 2 \\ 3 & 2 & -3 & 0\\ \end{array}$

Теперь для удобства и практичности нужно преобразовать матрицу так, чтобы в верхнем углу крайнего столбца была $1$.

Для этого к 1-ой строчке нужно прибавляем строчку из середины, умноженную на $-1$, а саму среднюю строчку записываем как есть, выходит:

$\begin{array}{ccc|c} -1 & -1 & 1 & -1 \\ 5 & 3 & -2 & 2 \\ 3 & 2 & -3 & 0\\ \end{array}$

Далее к средней строчке прибавим верхнюю, умноженную на $5$, а последнюю строчку преобразуем, умножив первую строчку на 3 и сложив с последней, получаем:

$\begin{array}{ccc|c} -1 & -1 & 1 & -1 \\ 0 & -2 & 3 & -3 \\ 0 & -1 & 0 & -3\\ \end{array}$

Домножим верхнюю и последнюю строчки на $-1$, а также поменяем местами последнюю и среднюю строки:

$\begin{array}{ccc|c} 1 & 1 & -1 & 1 \\ 0 & 1 & 0 & 3 \\ 0 & -2 & 3 & -3\\ \end{array}$

Далее сложим последнюю строчку с удвоенной средней:

$\begin{array}{ccc|c} 1 & 1 & -1 & 1 \\ 0 & 1 & 0 & 3 \\ 0 & 0 & 3 & 3\\ \end{array}$

И разделим последнюю строчку на $3$:

$\begin{array}{ccc|c} 1 & 1 & -1 & 1 \\ 0 & 1 & 0 & 3 \\ 0 & 0 & 1 & 1\\ \end{array}$

Получаем следующую систему уравнений, равносильную исходной:

$\begin{cases} x_1 + x_2 – x_3 = 1\\ x_2 = 3 \\ x_3 = 1 \end{cases}$

Из верхнего уравнения выражаем $x_1$:

$x1 = 1 + x_3 – x_2 = 1 + 1 – 3 = -1$.

Пример 2

Пример решения системы, заданной с помощью матрицы 4 на 4 методом Гаусса

$\begin{array}{cccc|c} 2 & 5 & 4 & 1 & 20 \\ 1 & 3 & 2 & 1 & 11 \\ 2 & 10 & 9 & 7 & 40\\ 3 & 8 & 9 & 2 & 37 \\ \end{array}$.

В начале меняем местами верхнюю исследующую за ней строчки, чтобы получить в левом верхнем углу $1$:

$\begin{array}{cccc|c} 1 & 3 & 2 & 1 & 11 \\ 2 & 5 & 4 & 1 & 20 \\ 2 & 10 & 9 & 7 & 40\\ 3 & 8 & 9 & 2 & 37 \\ \end{array}$.

Теперь умножим верхнюю строчку на $-2$ и прибавим ко 2-ой и к 3-ьей. К 4-ой прибавляем 1-ую строку, домноженную на $-3$:

$\begin{array}{cccc|c} 1 & 3 & 2 & 1 & 11 \\ 0 & -1 & 0 & -1 & -2 \\ 0 & 4 & 5 & 5 & 18\\ 0 & -1 & 3 & -1 & 4 \\ \end{array}$

Теперь к строке с номером 3 прибавляем строку 2, умноженную на $4$, а к строке 4 прибавляем строку 2, умноженную на $-1$.

$\begin{array}{cccc|c} 1 & 3 & 2 & 1 & 11 \\ 0 & -1 & 0 & -1 & -2 \\ 0 & 0 & 5 & 1 & 10\\ 0 & 0 & 3 & 0 & 6 \\ \end{array}$

Домножаем строку 2 на $-1$, а строку 4 делим на $3$ и ставим на место строки 3.

$\begin{array}{cccc|c} 1 & 3 & 2 & 1 & 11 \\ 0 & 1 & 0 & 1 & 2 \\ 0 & 0 & 1 & 0 & 2\\ 0 & 0 & 5 & 1 & 10 \\ \end{array}$

Теперь прибавляем к последней строке предпоследнюю, домноженную на $-5$.

$\begin{array}{cccc|c} 1 & 3 & 2 & 1 & 11 \\ 0 & 1 & 0 & 1 & 2 \\ 0 & 0 & 1 & 0 & 2\\ 0 & 0 & 0 & 1 & 0 \\ \end{array}$

Решаем полученную систему уравнений:

$\begin{cases} m = 0 \\ g = 2\\ y + m = 2\ \ x + 3y + 2g + m = 11\end{cases}$

$y=2$, $x = 0$.

Обратный ход метода Гаусса

Обратный ход метода Гаусса

Шаг 1. Из последнего уравнения системы (1.15) находим уr, подставив вместо свободных неизвестных произвольные числа tn-r:

Шаг 2. Подставляем найденный уr в предпоследнее уравнение и находим yr-1:

. . .

Шаг r. Подставляем найденные уr, …, у2 в первое уравнение находим у1:

В результате, получаем решение системы (1.11), в котором базисные переменные выражены через свободные переменные.

Замечание. Из доказательства теоремы Кронекера-Капелли следует, что:

· если rangA = rang = n , то система совместна и имеет единственное решение;

· если rangA = rang < n, то система совместна и имеет бесконечное множество решений;

· если rangA < rang , то система несовместна.

Пример 1.3. Решить систему линейных уравнений:

.

Решение. Приведем расширенную матрицу системы:

к трапециевидной форме. Поменяем местами первую и вторую строки матрицы, затем умножим элементы первой строки на –3 и прибавим к элементам второй сроки, элементы первой строки умножим на –2 и прибавим к элементам третьей строки, получим:

 

 
 

Полученная матрица не является трапециевидной, так как на главной диагонали есть элемент, равный нулю. Поменяем местами второй и третий столбцы матрицы, затем умножим элементы второй строки на –1 и прибавим к элементам третьей строки, получим:

Матрица имеет трапециевидную форму, причем в полученных матрицах по две ненулевых, строки, т.е. rang A = rang = 2, следовательно, по теореме Кронекера-Капелли система совместна и имеет бесконечное множество решений. Полученной матрице соответствует система уравнений, эквивалентная исходной:

где y1 = x1, y2 = x3, y3 = x2, (второй и третий столбцы в расширенной матрице менялись местами). Пусть у3 = t, тогдаиз второго уравнения находим у2 = 2,5 и, подставляя у2 в первое уравнение, получим у1 = –3,5 – t. Таким образом, решением данной системы уравнений будут : х1 = –3,5 – t, х2 = t, x3 = 2,5.


Дата добавления: 2015-07-15; просмотров: 100 | Нарушение авторских прав


Читайте в этой же книге: Определители | Свойства определителей | Доказательство свойств определителей | Системы линейных уравнений. Формулы Крамера | Теорема 1.1 (Крамера) | Действия над матрицами | Ранг матрицы | Утверждение 1.4 | Метод Гаусса. Теорема Кронекера-Капелли | Прямой ход метода Гаусса |
mybiblioteka.su – 2015-2021 год. (0.02 сек.)

Обращение матрицы с использованием элементарных операций со строками (Гаусс-Джордан)

Также называется методом Гаусса-Жордана.

Это интересный способ найти обратную матрицу:

Поиграйте со строками (сложение, умножение или замена) пока мы не превратим Matrix A в Identity Matrix I

И ТАКЖЕ внесение изменений в матрицу идентичности, она волшебным образом превращается в инверсию!

«Элементарные операции со строками» – это простые вещи, такие как добавление строк, умножение и замена местами… но давайте посмотрим на примере:

Пример: найти обратную букву “А”:

Мы начинаем с матрицы A и записываем ее с Матрицей идентичности I рядом с ней:


(это называется «Расширенная матрица»)

Идентификационная матрица

«Матрица идентичности» является матричным эквивалентом числа «1»:


Матрица идентификации 3×3

  • Это «квадрат» (в нем столько же строк, что и столбцов),
  • Он имеет 1 с по диагонали и 0 с по всей остальной части.
  • Его символ – заглавная буква I .

Теперь мы делаем все возможное, чтобы превратить «А» (Матрица слева) в Матрицу Идентичности. Цель состоит в том, чтобы матрица A имела 1 с по диагонали и 0 с в другом месте (матрица идентичности) … и правая сторона используется для поездки, и все операции, выполняемые с ней, тоже.

Но мы можем выполнять только эти «Элементарные операции со строками» :

  • поменять местами строк
  • умножить или разделить каждый элемент в строке на константу
  • заменить строку на , добавив или вычтя из нее несколько другой строки

И мы должны сделать это для всей строки , вот так:

Начните с A рядом с I

Добавить строку 2 к строке 1,

, затем разделите строку 1 на 5,

Затем возьмите 2 раза первую строку и вычтите ее из второй строки,

Умножить вторую строку на -1/2,

Теперь поменяйте местами вторую и третью строки,

Наконец, вычтите третью строку из второй строки,

И готово!

И матрица была преобразована в матрицу идентичности…

… и в то же время идентификационная матрица превратилась в A -1

СДЕЛАНО! Как по волшебству, и так же весело, как решать любую головоломку.

И обратите внимание: не существует “правильного способа” сделать это, просто продолжайте играть, пока у нас не получится!

(Сравните этот ответ с тем, который мы получили по вопросу об обратной матрице с использованием младших, сомножителей и адъюгата. Это то же самое? Какой метод вы предпочитаете?)

Большие матрицы

Мы можем сделать это с матрицами большего размера, например, попробуйте эту матрицу 4×4:

Начать как это:

Посмотри, сможешь ли ты сделать это сам (я бы начал с деления первого ряда на 4, но ты делаешь это по-своему).

Вы можете проверить свой ответ с помощью калькулятора матрицы (используйте кнопку «inv (A)»).

Почему это работает

Мне нравится думать об этом так:

  • когда мы превращаем «8» в «1» путем деления на 8,
  • и проделайте то же самое с «1», он превратится в «1/8»

И “1/8” является (мультипликативным) , обратным 8

Или, точнее говоря:

Общий эффект всех операций со строками такой же, как , умноженный на A -1

Итак, A становится I (потому что A -1 A = I )
And I становится A -1 (потому что A -1 I = A -1 )

scipy.stats.invgauss – Руководство SciPy v1.7.1

rvs (mu, loc = 0, scale = 1, size = 1, random_state = None)

Случайные переменные.

pdf (x, mu, loc = 0, scale = 1)

Функция плотности вероятности.

logpdf (x, mu, loc = 0, scale = 1)

Логарифм функции плотности вероятности.

cdf (x, mu, loc = 0, scale = 1)

Кумулятивная функция распределения.

logcdf (x, mu, loc = 0, scale = 1)

Журнал интегральной функции распределения.

sf (x, mu, loc = 0, scale = 1)

Функция выживания (также определяется как 1 - cdf , но sf иногда более точна).

logsf (x, mu, loc = 0, scale = 1)

Журнал функции выживания.

ppf (q, mu, loc = 0, scale = 1)

Функция процентной точки (обратная cdf – процентили).

isf (q, mu, loc = 0, scale = 1)

Обратная функция выживания (обратная sf ).

момент (n, mu, loc = 0, scale = 1)

Нецентральный момент заказа n

статистика (mu, loc = 0, scale = 1, моменты = ’mv’)

Среднее (m), дисперсия (v), перекос (s) и / или эксцесс (k).

энтропия (мю, loc = 0, масштаб = 1)

(Дифференциальная) энтропия RV.

подходит (данные)

Оценка параметров для общих данных. См. Scipy.stats.rv_continuous.fit для подробной документации аргументов ключевого слова.

expect (func, args = (mu,), loc = 0, scale = 1, lb = None, ub = None, conditional = False, ** kwds)

Ожидаемое значение функции (одного аргумента) относительно распределения.

медиана (mu, loc = 0, scale = 1)

Медиана распределения.

среднее (mu, loc = 0, scale = 1)

Среднее значение распределения.

var (mu, loc = 0, scale = 1)

Вариация распределения.

std (mu, loc = 0, scale = 1)

Стандартное отклонение распределения.

интервал (альфа, мю, loc = 0, масштаб = 1)

Конечные точки диапазона, который содержит дробную часть альфа [0, 1] распределения

Форма исключения по Гауссу и рядового эшелона

Обратное преобразование матрицы – не совсем простая задача, если вы еще не знакомы с методом исключения Гаусса. Обратная матрица используется в большом количестве алгоритмов, одним из самых простых является линейная регрессия.

Есть два шага для инвертирования матрицы:

  1. Проверка обратимости матрицы путем нахождения Определителя
  2. Инвертирование матрицы с помощью варианта исключения Гаусса Гаусса-Джордана
  1. Проверка, является ли матрица обратимой
  2. Инвертирование матрицы
  3. Список литературы

Проверка обратимости матрицы

Понятие ранга в линейной алгебре относится к размерности. Все матрицы имеют некоторый размер $ m \ times n $, где $ m $ – количество строк, а $ n $ – количество столбцов.

Ранг матрицы

  1. Ранг $ 1 $: когда матрица является строкой, она имеет ранг $ 1 $.
  2. Ранг $ 2 $: Когда матрица представляет собой плоскость, она имеет ранг $ 2 $.
  3. Считается, что матрица имеет полный ранг тогда и только тогда, когда матрица не содержит столбец в виде линейной комбинации двух столбцов, потому что это означает

Любая заданная матрица может иметь только такой высокий ранг, как количество столбцы $ n $ для этой матрицы. Но матрица с $ n = 3 $ может иметь только ранг $ 2 $, потому что некоторое преобразование может сжимать векторы-столбцы на плоскости – и ранг может даже быть уменьшен до ранга $ 1 $, если преобразование сжимает векторы на прямой линия.

Давайте разберемся, что это значит.

Линейные комбинации

Мы определяем три вектора $ \ vec {v} $, $ \ vec {u} $ и $ \ vec {w} $ и объединяем их как векторы-столбцы в матрице $ A $.

$$ \ vec {v} = \ begin {bmatrix} 2 \\ 0 \\ 1 \ end {bmatrix} , \ quad \ vec {u} = \ begin {bmatrix} 1 \\ 2 \\ 1 \ end {bmatrix} , \ quad \ vec {w} = \ begin {bmatrix} 1 \\ -2 \\ 0 \ end {bmatrix} , \ quad А = \ begin {bmatrix} 2 и 1 и 1 \\ 0 и 2 и -2 \\ 1 и 1 и 0 \ end {bmatrix} $$

Если мы выполним сложение векторов над $ \ vec {u} $ и $ \ vec {w} $, то результатом будет вектор $ \ vec {v} $.Это означает, что $ \ vec {v} $ является линейной комбинацией $ \ vec {u} $ и $ \ vec {w} $, т.е. матрица $ A $ не имеет полного ранга.

$$ \ vec {u} + \ vec {w} = \ begin {bmatrix} 1 + 1 \\ 2 + (- 2) \\ 1 + 0 \ end {bmatrix} знак равно \ begin {bmatrix} 2 \\ 0 \\ 1 \ end {bmatrix} $$

Аналогичным образом, если вы вычтете $ \ vec {v} $ и $ \ vec {u} $, то обнаружите, что они являются линейной комбинацией $ \ vec {w} $.

Геометрическая интерпретация линейных комбинаций состоит в том, что мы теряем информацию , если бы мы использовали это конкретное преобразование, связанное с матрицей $ A $, потому что результирующая матрица преобразования больше не охватывает все три измерения пространства.

Вышеупомянутая матрица $ A $ имеет ранг $ 2 $, и, как мы увидим позже, мы можем вычислить ранг матрицы, пытаясь уменьшить матрицу до сокращенного эшелона строк формы . Но пока мы можем определить, можно ли инвертировать матрицу, вычислив определитель.

Вычисление определителя

Когда определитель матрицы равен нулю, ранг матрицы не является полным рангом, что означает, что мы не можем инвертировать матрицу. Точно так же есть 23 других свойства, которые вы можете использовать для проверки обратимости матрицы.

Для матрицы 3×3 следующая формула:

$$ \ begin {align} det (А) знак равно det \ left (\ begin {bmatrix} а & б & с \\ d & e & f \\ g & h & i \ end {bmatrix} \ right) \\ & = aei + bfg + cdh – ceg – bdi – afh \ end {align} $$

Итак, давайте подставим число и найдем решение.

$$ \ begin {align} det (А) знак равно det \ left (\ begin {bmatrix} 2 и 1 и 1 \\ 0 и 2 и -2 \\ 1 и 1 и 0 \ end {bmatrix} \ right) \\ & = 2 * 2 * 0 + 1 * (- 2) * 1 + 1 * 0 * 1 – 1 * 2 * 1 – 1 * 0 * 0 – 2 * (- 2) * 1 \\ & = 0 \ end {align} $$

Вот и все, матрица $ A $ необратима, так как определитель равен нулю, хотя, если бы у нас была обратимая матрица, определитель был бы ненулевым.Другими словами, мы не можем найти обратную матрицу $ A $, но если бы вместо этого у нас была другая матрица $ B $, где $ det (B) \ ne 0 $, мы смогли бы найти обратную.

Обратите внимание, что вычисление определителя возможно только для квадратных матриц, то есть матриц, у которых количество строк $ m $ равно количеству столбцов $ n $.

Инвертирование матрицы

После того, как мы проверили, является ли матрица обратимой, и выяснилось, что она обратима, мы можем продолжить и использовать метод исключения Гаусса, чтобы инвертировать матрицу.В частности, этот метод называется методом исключения Гаусса-Жордана, который представляет собой вариант алгоритма исключения Гаусса.

Инициализация алгоритма

Следующая матрица $ B $ обратима, поскольку определитель матрицы не равен нулю. Мы будем использовать матрицу в качестве примера.

$$ B = \ begin {bmatrix} 1 и 5 и -1 \\ 2 и 2 и -2 \\ -1 и 4 и 3 \ end {bmatrix} $$

Мы инициализируем алгоритм, собирая матрицу $ B $ с левой стороны, разделенную вертикальной линией, а затем единичную матрицу для $ B $, обозначенную $ I $.Единичная матрица – это матрица с единицей, проходящей по диагонали от верхнего левого угла к нижнему правому (главная диагональ), и нулями в остальных элементах.

$$ [B | I] = \ left [ \ begin {array} {ccc | ccc} 1 и 5 и -1 и 1 и 0 и 0 \\ 2 & 2 & -2 & 0 & 1 & 0 \\ -1 и 4 и 3 и 0 и 0 и 1 \ end {массив} \верно] $$

При выполнении операций мы выполняем их как с матрицей $ B $, так и с $ I $. После многих операций матрица B будет преобразована в форму эшелона строк, а единичная матрица $ I $ будет обратной матрице $ B $.

Цель исключения Гаусса-Джордана – преобразовать матрицу в сокращенную форму строк .

Давайте на минуту разберемся, что это значит.

Форма сокращенного эшелона строк

Чтобы матрица была в форме сокращенного эшелона строк, она должна удовлетворять следующим условиям:

  1. Все записи в строке должны быть равны 0 $ до первого появления числа $ 1 $ .
  2. Первый $ 1 $ в строке всегда находится справа от первого $ 1 $ в строке выше.
  3. Все строки, которые состоят только из $ 0 $, помещаются под строками, в которых их нет.
  4. Первый $ 1 $ в строке содержит только $ 0 $ в той же записи во всех строках выше $ 1 $. Другими словами, первый $ 1 $ в строке содержит только $ 0 $ в том же столбце для всех строк выше $ 1 $.

Эти условия подразумевают, что строка под первым $ 1 $ в любой заданной строке содержит $ 0 $ ниже и слева от этого первого $ 1 $.

Вот примеры матриц, которые находятся в сокращенной форме эшелона строк:

$$ А = \ begin {bmatrix} 1 & 0 & 0 \\ 0 & 1 & 0 \\ 0 и 0 и 1 \ end {bmatrix} , \ quad B = \ begin {bmatrix} 1 & -2 & 0 & 0 \\ 0 & 0 & 1 & 0 \\ 0 & 0 & 0 & 1 \ end {bmatrix} , \ quad C = \ begin {bmatrix} 1 & 0 & -2 & 0 & 3 \\ 0 и 1 и 3 и 0 и 5 \\ 0 и 0 и 0 и 1 и 6 \ end {bmatrix} $$

Вот примеры матриц, которые НЕ находятся в форме сокращенного эшелона строк:

$$ А = \ begin {bmatrix} 2 и 1 и 0 \\ 0 & 0 & 1 \\ 0 & 0 & 0 \ end {bmatrix} , \ quad B = \ begin {bmatrix} 1 & 0 & 0 \\ 0 & 1 & 0 \\ 0 и 1 и 0 \ end {bmatrix} , \ quad C = \ begin {bmatrix} 1 & 0 & 0 \\ 0 & 0 & 0 \\ 0 и 1 и 0 \ end {bmatrix} , \ quad D = \ begin {bmatrix} 1 и 7 и 0 \\ 0 и 1 и -2 \\ 0 и 0 и 1 \ end {bmatrix} $$

Обратите внимание, что матрица в форме сокращенного эшелона строк также имеет форму эшелона строк.

Обозначения и операции

Ниже приведены операции, которые мы можем выполнить во время этого процесса исключения, чтобы преобразовать матрицу в сокращенную форму эшелона строк.

Как правило, принятые обозначения для выполнения операций со строками выглядят следующим образом:

$$ R _ {\ text {некоторый номер строки x}} \ rightarrow R _ {\ text {некоторый другой номер строки y}} $$

Который означает, что $ R_x $ становится равным строке $ R_y $. Но это не так просто, потому что мы можем использовать три основные операции со строками, которые могут быстро запутать.

  1. Добавление строки: мы можем добавить одну строку к другой строке, например,
    $ R_3 \ rightarrow R_2 + R_3 $.
    Обратите внимание, что это также означает, что мы можем выполнять вычитание,
    например $ R_3 \ rightarrow -R_2 + R_3 $.
  2. Умножение строк: мы можем добавить или вычесть ту же строку на некоторую величину, например,
    $ R_3 \ rightarrow 2R_2 – 1 / 4R_1 + R_3 $.
    Обратите внимание, что мы также можем выполнять деление при умножении.
  3. Перестановка строк: мы можем поменять местами строки в матрице, чтобы они занимали место друг друга, например.г. $ R_3 \ rightarrow R_2 $ и $ R_2 \ rightarrow R_3 $.

Вы можете даже подумать об очень простом умножении или делении строк, когда вы делите строку на 2 доллара или умножаете строку на 3 доллара.

При выполнении этих матричных операций вручную важно содержать ее в чистоте и порядке, иначе вы быстро потеряете из виду, что делали. Всю арифметику сложно уследить.

Полный пример

Наконец-то у нас есть все воедино, мы понимаем, что такое сокращенная форма эшелона строк, и мы знакомы с операциями, которые мы можем выполнить, чтобы преобразовать матрицу в форму сокращенного эшелона строк.

А теперь пора взглянуть на пример и применить то, что мы только что узнали. То есть применить алгоритм Гаусса-Жордана для инвертирования матрицы. Помните, что цель состоит в том, чтобы преобразовать матрицу в форму сокращенного эшелона строк, что означает, что матрица должна удовлетворять определенным условиям, перечисленным в разделе Форма сокращенного эшелона строк.

$$ [B | I] = \ left [ \ begin {array} {ccc | ccc} 1 и 5 и -1 и 1 и 0 и 0 \\ 2 & 2 & -2 & 0 & 1 & 0 \\ -1 и 4 и 3 и 0 и 0 и 1 \ end {массив} \верно] $$

Целью первых нескольких операций является преобразование первого столбца таким образом, чтобы только $ 0 $ находились ниже первого $ 1 $.

  1. Во-первых, мы добавляем $ -2R_1 $ к $ R_2 $
  2. Во-вторых, мы добавляем $ R_1 $ к $ R_3 $

$$ \левый[ \ begin {array} {ccc | ccc} 1 и 5 и -1 и 1 и 0 и 0 \\ 2 & 2 & -2 & 0 & 1 & 0 \\ -1 и 4 и 3 и 0 и 0 и 1 \ end {массив} \верно] R_2 \ стрелка вправо -2R_1 + R_2 \левый[ \ begin {array} {ccc | ccc} 1 и 5 и -1 и 1 и 0 и 0 \\ 0 & -8 & 0 & -2 & 1 & 0 \\ -1 и 4 и 3 и 0 и 0 и 1 \ end {массив} \верно] $$

$$ \левый[ \ begin {array} {ccc | ccc} 1 и 5 и -1 и 1 и 0 и 0 \\ 0 & -8 & 0 & -2 & 1 & 0 \\ -1 и 4 и 3 и 0 и 0 и 1 \ end {массив} \верно] R_3 \ rightarrow R_1 + R_3 \левый[ \ begin {array} {ccc | ccc} 1 и 5 и -1 и 1 и 0 и 0 \\ 0 & -8 & 0 & -2 & 1 & 0 \\ 0 и 9 и 2 и 1 и 0 и 1 \ end {массив} \верно] $$

Для второй строки мы видим, что в строке нет $ 1 $, поэтому мы должны это изменить.А во-вторых, нам также нужно, чтобы внизу и слева от первого $ 1 $ во второй строке были только $ 0 $.

  1. Во-первых, мы делим $ R2 $ на $ -8 $
  2. Во-вторых, мы прибавляем $ -9R_2 $ к $ R_3 $

$$ \левый[ \ begin {array} {ccc | ccc} 1 и 5 и -1 и 1 и 0 и 0 \\ 0 & -8 & 0 & -2 & 1 & 0 \\ 0 и 9 и 2 и 1 и 0 и 1 \ end {массив} \верно] R_2 \ rightarrow \ frac {1} {- 8} R_2 \левый[ \ begin {array} {ccc | ccc} 1 и 5 и -1 и 1 и 0 и 0 \\ 0 и 1 и 0 и 0.25 & -0,125 & 0 \\ 0 и 9 и 2 и 1 и 0 и 1 \ end {массив} \верно] $$

$$ \левый[ \ begin {array} {ccc | ccc} 1 и 5 и -1 и 1 и 0 и 0 \\ 0 & 1 & 0 & 0,25 & -0,125 & 0 \\ 0 и 9 и 2 и 1 и 0 и 1 \ end {массив} \верно] R_3 \ rightarrow -9R_2 + R_3 \левый[ \ begin {array} {ccc | ccc} 1 и 5 и -1 и 1 и 0 и 0 \\ 0 & 1 & 0 & 0,25 & -0,125 & 0 \\ 0 & 0 & 2 & -1.25 и 1,125 и 1 \ end {массив} \верно] $$

Затем мы позаботимся о последнем столбце, в котором также нет $ 1 $. Мы делаем это, разделив $ R_3 $ на $ 2 $.

$$ \левый[ \ begin {array} {ccc | ccc} 1 и 5 и -1 и 1 и 0 и 0 \\ 0 & 1 & 0 & 0,25 & -0,125 & 0 \\ 0 & 0 & 2 & -1,25 & 1,125 & 1 \ end {массив} \верно] R_3 \ rightarrow \ frac {1} {2} R_3 \левый[ \ begin {array} {ccc | ccc} 1 и 5 и -1 и 1 и 0 и 0 \\ 0 и 1 и 0 и 0.25 & -0,125 & 0 \\ 0 и 0 и 1 и -0,625 и 0,5625 и 0,5 \ end {массив} \верно] $$

Напомню правило 4 из раздела «Форма сокращенного эшелона строк».

Первый $ 1 $ в строке содержит только $ 0 $ в той же записи во всех строках выше $ 1 $. Другими словами, первый $ 1 $ в строке содержит только $ 0 $ в том же столбце для всех строк выше $ 1 $.

Это означает, что нам нужно избавиться от $ 5 $ и $ -1 $ в первой строке. В следующих шагах мы также преобразуем некоторые записи в дроби, а не в десятичные числа, чтобы они лучше отображались на экране.

  1. Во-первых, мы добавляем $ R_3 $ к $ R_1 $
  2. Во-вторых, мы добавляем $ -5R_2 $ к $ R_1 $

$$ \левый[ \ begin {array} {ccc | ccc} 1 и 5 и -1 и 1 и 0 и 0 \\ 0 & 1 & 0 & 0,25 & -0,125 & 0 \\ 0 и 0 и 1 и -0,625 и 0,5625 и 0,5 \ end {массив} \верно] R_1 \ rightarrow R_3 + R_1 \левый[ \ begin {array} {ccc | ccc} 1 & 5 & 0 & \ frac {3} {8} & \ frac {9} {16} & \ frac {1} {2} \\ 0 & 1 & 0 & \ frac {1} {4} & – \ frac {1} {8} & 0 \\ 0 & 0 & 1 & – \ frac {5} {8} & \ frac {9} {16} & \ frac {1} {2} \ end {массив} \верно] $$

$$ \левый[ \ begin {array} {ccc | ccc} 1 & 5 & 0 & \ frac {3} {8} & \ frac {9} {16} & \ frac {1} {2} \\ 0 & 1 & 0 & \ frac {1} {4} & – \ frac {1} {8} & 0 \\ 0 & 0 & 1 & – \ frac {5} {8} & \ frac {9} {16} & \ frac {1} {2} \ end {массив} \верно] R_1 \ rightarrow -5R_2 + R_1 \левый[ \ begin {array} {ccc | ccc} 1 & 0 & 0 & – \ frac {7} {8} & 1 \ frac {3} {16} & \ frac {1} {2} \\ 0 & 1 & 0 & \ frac {1} {4} & – \ frac {1} {8} & 0 \\ 0 & 0 & 1 & – \ frac {5} {8} & \ frac {9} {16} & \ frac {1} {2} \ end {массив} \верно] $$

И все, мы преобразовали матрицу $ B $ в сокращенную форму эшелона строк, что означает, что единичная матрица $ I $ была преобразована в обратную матрицу $ B $.

$$ обратный (B) = \ begin {bmatrix} – \ frac {7} {8} & 1 \ frac {3} {16} & \ frac {1} {2} \\ \ frac {1} {4} & – \ frac {1} {8} & 0 \\ – \ frac {5} {8} & \ frac {9} {16} & \ frac {1} {2} \ end {bmatrix} знак равно \ begin {bmatrix} -0,875 и 1,1875 и 0,5 \\ 0,25 и -0,125 и 0 \\ -0,625 и 0,5625 и 0,5 \ end {bmatrix} $$


Обладая базовыми навыками программирования, вы можете использовать Python с пакетом NumPy для довольно простого выполнения обратной операции.

  импортировать numpy как np

B = np.array ([[1,5, -1], [2,2, -2], [- 1,4,3]])

inv = np.linalg.inv (B)

печать (инв.)
  

Это дает тот же результат, который мы получили от наших операций со строками, и может быть хорошим инструментом для проверки ваших результатов при выполнении исключения вручную.

  массив ([[- 0,875, 1,1875, 0,5],
       [0,25, -0,125, 0],
       [-0,625, 0,5625, 0,5]])
  

Ссылки

Хорошее видео, чтобы понять общую концепцию рангов, определителей и обратных матриц:

Еще один пример того, как найти обратную матрицу, предоставленную Академией Хана:

Часть 7: Инверсии и Гаусс -Jordan Elimination | Авниш | Линейная алгебра

Предполагая матрицу A.

Матрица, обратная матрице A, будет другой матрицей того же порядка, которая при умножении на A вернет I (матрицу идентичности) в качестве произведения.

Инверсия к A обозначается буквой, представляющей матрицу в степени (-1).

Обратная матрица A

Таким образом, что

Идентификационная матрица имеет тот же порядок

Умножение матриц не коммутативно, но когда мы умножаем матрицу на ее обратное умножение, может быть коммутативным.

Порядок не имеет значения, когда матрица умножается на обратную.

Кроме того, только квадратные матрицы имеют обратные значения .

Деление матриц в части 2 не обсуждалось.

Теперь с помощью инверсий можно производить деление двух матриц.

Возьмем пример

Это эквивалентно умножению матрицы B на матрицу, обратную A.

Как

Тогда

Результирующая матрица

Алгоритм для поиска обратной матрицы, он похож на метод исключения Гаусса или мы можем сказать это метод исключения Гаусса, расширенный еще на один шаг.

Он назван в честь Карла Фридриха Гаусса и Вильгельма Йордана, немецкого геодезиста.

Вильгельм Джордан улучшил стабильность алгоритма.

Карл Фридрих Гаусс Вильгельм Джордан

Чтобы выполнить метод исключения Гаусса-Жордана, мы должны:

1. Составить расширенную матрицу из данной матрицы и ее единичной матрицы (порядок матрицы идентичности определяется в соответствии с порядком данной матрицы).

2. Выполните исключение (как на шаге 2 исключения Гаусса), стремясь получить форму эшелона строк в левой половине расширенной матрицы.

3. Уменьшите его еще больше, чтобы получить Reduced Row Echelon Form (Identity matrix) в левой половине расширенной матрицы.

4. Правая половина расширенной матрицы является обратной для данной матрицы.

Предположим, что мы должны найти обратную матрицу A (см. Выше) с помощью метода исключения Гаусса-Жордана.

Шаг 1 (Создание расширенной матрицы):

Матрица AI Матрица идентичности для матрицы A Расширенная матрица матриц A и I

Шаг 2 (Исключение):

Применение операций

Элементы, удаленные из первого столбца

Теперь применим аналогичные операции для столбца 2.

Row echelon form

Если бы мы выполняли функцию исключения по Гауссу, мы бы уже закончили и начали обратную замену, но чтобы найти обратную, мы должны сделать еще один шаг вперед. Нам нужно найти форму сокращенного эшелона строк

Шаг 3 (исключение в форму сокращенного эшелона строк):

Для получения формы сокращенного эшелона строк мы стремимся преобразовать левую половину расширенной матрицы в единичную матрицу, используя аналогичные операции как в методе исключения Гаусса, и мы начинаем снизу вверх.

Сначала мы приводим строку 3 к форме единичной матрицы.

Затем мы уменьшаем строку 2.

Вторая строка сокращается

Теперь мы уменьшаем строку 1, и мы получим уменьшенную форму эшелона строк.

Форма сокращенного эшелона строк

Шаг 4 (отдельная правая половина расширенной матрицы):

Правая половина расширенной матрицы является обратной матрицей A.

Обратной матрицей A

Нет, матрица должна удовлетворять следующим двум условиям быть обратимым.

1. Это должна быть квадратная матрица.

2. Матрица не должна быть сингулярной (ее определитель не равен 0).

Нахождение обратной матрицы с помощью метода исключения Гаусса-Жордана и присоединенной матрицы | by Pollux Rey

Мы знаем, что

Если определитель матрицы коэффициентов A , det (A) , не равен нулю, то A имеет обратное значение. Если A имеет инверсию, и если мы умножим ее на приведенное выше уравнение, то получим

Это означает, что мы можем найти решение для системы, используя обратную матрицу, при условии, что B дано .В этой статье мы представим два метода, чтобы получить его: Гаусс-Джордан Элиминг и метод сопряженной матрицы .

Исключение Гаусса-Джордана

Выше мы видели, что когда A умножается на его обратное, получается единичная матрица I (группа единиц на главной диагонали матрицы и окруженная с нулями). Математически:

. Если мы поменяем местами положение A и его обратное,

Если подумать сложнее, это будет похоже на решение для X .{-1} . Ранее мы увеличили A и B и использовали исключение Гаусса-Джордана, чтобы получить X . Это то, что мы здесь и сделаем: мы сложим A и I вместе и уменьшим его в ряд, чтобы получить обратное значение A .

Давайте сначала сформулируем некоторые термины.

Ранее мы упоминали, как получить определитель матрицы, но теперь мы углубимся в то, из чего состоит определитель.

Минор матрицы

Минор матрицы , M_ {i, j} , это определитель, полученный при игнорировании значений в строке i и j -й столбец вашей матрицы.

Кофактор матрицы

Кофактор матрицы , C_ {i, j} , является младшим со знаком . Он следует формуле

Определитель матрицы

Предположим, что у нас есть квадратная матрица

Определитель A равен

, если мы хотим развернуть i -ю строку, или

если мы хотим развернуть j -й столбец.Расширение C_ {i, j} приведет к

или

Чтобы лучше понять три концепции, давайте ответим на следующий вопрос:

(Элементарная линейная алгебра Рона Ларсона, 8-е издание, стр. 116)
Найдите миноры и кофакторы матрицы

Сопряжение матрицы

Транспонирование матрицы сомножителей называется сопряженным элементом матрицы , то есть когда вы создаете новую матрицу, и значения этой матрицы являются кофакторами для каждого значение в старой матрице, затем вы помещаете значения первой строки полученной матрицы в первый столбец и так далее.Математически говоря,

Как получить обратную матрицу для этого метода?

Рассмотрим произведение A и adj (A) ,

Если мы умножим выделенную строку и столбец, показанные на фотографии ниже,

, вы получите

Теперь давайте выделим средняя строка в A и средний столбец в B

Единственный способ снова получить det (A) , когда i = j .В противном случае он равен 0 (см. Разложение детерминант Лапласа). Итак, если это так, то основная диагональ произведения A и adj (A) равна

, используя приведенное выше уравнение, мы получим

Итак, для этого метода, чтобы получить матрица, обратная матрице, мы должны получить ее сопряженный элемент и разделить его на его определитель.

Геомагнитная инверсия Гилберта-Гаусса, зафиксированная в плиоценовых вулканических толщах Грузии (Малый Кавказ): повторное посещение | Земля, планеты и космос

  • Bogue, S.У. и Х. А. Пол, Отличительное поведение поля после геомагнитных инверсий, Geophys. Res. Lett. , 20 , 2399–2402, 1993.

    Статья Google Scholar

  • Кэмпс, П., Г. Руффе, В. П. Щербаков, В. В. Щербакова, М. Прево, А. Муссин-Пушкин, Л. Шолпо, А. Гогуитчайшвили, Б. Асанидзе, Палеомагнитные и геокронологические исследования геомагнитного поля. разворот или экскурсия, зафиксированная в плиоценовых вулканических породах Грузии (Малый Кавказ), Phys.Планета Земля. Интер. , 96 , 41–59, 1996.

    Статья Google Scholar

  • Каркайе, Дж. Т., Д. Бурлес и Н. Тувени, Геомагнитный дипольный момент и интеркалибрация скорости производства 10 Be из аутигенного 10 Be / 9 Be за последние 1,3 млн лет, Geochem. Геофиз., Геосист. , 5 , doi 1029 / 2003GC000641, 2004.

  • Коэ, Р., Палеонапряженность магнитного поля Земли, определенная по третичным и четвертичным породам, Дж.Geophys. Res. , 83 , 1740–1756, 1967.

    Статья Google Scholar

  • Коу, Р. С., С. Громме и Э. А. Манкинен, Геомагнитные палеонапряженности от радиоуглеродных лавовых потоков на Гавайях и вопрос о недипольной депрессии Тихого океана, J. Geophys. Res. , 83 , 1740–1756, 1978.

    Артикул Google Scholar

  • Кокс, А., Длины интервалов геомагнитной полярности, J. Geophys. Res. , 73 , 3247–3260, 1968.

    Статья Google Scholar

  • Дэй Р., Фуллер М., Шмидт В. Гистерезисные свойства титаномагнетитов: зависимость от размера зерен и состава, Phys. Планета Земля. Интер. , 13 , 260–267, 1977 г.

    Статья Google Scholar

  • Данлоп, Д.J., Теория и применение графика Дня (Mrs / Ms по сравнению с Hcr / Hc), Теоретические кривые и тесты с использованием данных о титаномагнетите, J. Geophys. Res. , 107 , DOI: 10.1029 / 2001JB000486, 2002.

  • Dunlop, D. and Ö. Оздемир, Рок-магнетизм, основы и границы , 573 стр., Cambridge University Press, 1997.

    Книга Google Scholar

  • Гогуитчаишвили А., Сологачвили Д., М.Прево, М. Кальво, Э. С. Павленичвили, Г. Майсурадзе, Э. Шнепп, Палеомагнитное и рок-магнитное исследование вулканического разреза плиоцена на юге Грузии (Кавказ), Геол. Mijnbouw , 76 , 135–143, 1997.

    Статья Google Scholar

  • Goguitchaichvili, A., M. Prevot, J. M. Dautria и M. Bacia, Термодетритовые и кристаллические намагниченности в исландском гиалокластите, J. Geophys. Res., 104 , 29219–29239, 1999a.

    Артикул Google Scholar

  • Goguitchaichvili, A., M. Prevot, N. Roberts, J. Thompson, Попытка определить абсолютную напряженность геомагнитного поля на юго-западе Исландии во время инверсии Гаусса-Матьямы, Phys. Планета Земля. Интер. , 115 , 53–66, 1999b.

    Артикул Google Scholar

  • Гогуитчаишвили, А., М. Прево и П. Кэмпс, Нет свидетельств сильных полей во время исландских геомагнитных инверсий R3-N3, Earth Planet. Sci. Lett. , 167 , 15–34, 1999c.

    Артикул Google Scholar

  • Goguitchaichvili, A., J. Morales, J. Urrutia-Fucugauchi, Об использовании термомагнитных кривых в палеомагнетизме, C. R. Acad. Наук, Планета Земля. Sci. , 333 , 699–704, 2001a.

    Google Scholar

  • Гогуитчаишвили, А., П. Кэмпс, Дж. Уррутия-Фукугаучи, Об особенностях геодинамо после разворотов или экскурсий, Phys. Планета Земля. Интер. , 124 , 81–93, 2001b.

    Артикул Google Scholar

  • Граттон М. и Дж. Шоу, Абсолютные вариации палеонапряженности во время предвестника перехода Матуяма-Брюнес, зарегистрированные в чилийских лавах, Планета Земля. Sci. Lett. , 162 (1–2), 61–72, 2007.

    Артикул Google Scholar

  • Губбинс Д., Различие между геомагнитными экскурсиями и инверсиями, Geophys. J. Int. , 137 , F1 – F3, 1999.

    Артикул Google Scholar

  • Хартл П. и Л. Токс, предшественник перехода Матуяма / Брюнес – нестабильность поля, зафиксированная в пелагических отложениях, Earth Planet. Sci.Lett. , 138 , 121–135, 1996.

    Статья Google Scholar

  • Эрреро-Бервера, Э. и Дж. П. Валет, Абсолютная палеонапряженность и записи инверсии из последовательности Вайнанаэ (Оаху, Гавайи, США), Планета Земля. Sci. Lett. , DOI: 10.1016 / j.epsl.2005, 2005.

    Google Scholar

  • Кент, Д. В. и Д. А. Шнайдер, Корреляция записей вариаций палеонапряженности в интервале перехода полярности Брюнес / Матуяма, Earth Planet.Sci. Lett. , 129 , 135–142, 1995.

    Статья Google Scholar

  • Киршвинк, Дж. Л., Линия и плоскость наименьших квадратов и анализ палеомагнитных данных, Geophys. J. R. Astron. Soc. , 62 , 699–718, 1980.

    Статья Google Scholar

  • Кнудсен, М. Ф., Н. Абрахамсен и П. Риисагер, Палеомагнитные данные из базальтов островов Зеленого Мыса для полностью обратных экскурсий в Brunhes Chron, Earth Planet.Sci. Lett. , 206 , 199–214, 2003.

    Статья Google Scholar

  • Костеров А., М. Перрин, Дж. М. Глен и Р. С. Коу, Палеонапряженность магнитного поля Земли в раннемеловое время: Базальт Парана, Бразилия, J. Geophys. Res. , 103 , 9739–9753, 1998.

    Статья Google Scholar

  • Laj, C., A. Mazaud, R.Уикс и Э. Эрреро-Бервера, Пути изменения геомагнитного поля, Nature , 351 , 347–350, 1991.

    Article Google Scholar

  • Леви С. Влияние размера частиц магнетита на определение палеонапряженности геомагнитного поля, Phys. Планета Земля. Интер. , 13 , 245–258, 1977 г.

    Статья Google Scholar

  • Майсурадзе, Г., Антропоген Антикавказа, Палеогр. Палеоклим. Палеоэкол. , 72 , 53–61, 1989.

    Статья Google Scholar

  • Манкинен, Э. А., М. Прево, С. С. Громме и Р. Коу, Переход геомагнитной полярности горы Стинс (Орегон) 1, Направленная история, продолжительность эпизодов и магнетизм горных пород, J. Geophys. Res. , 90 , 10393–10416, 1985.

    Артикул Google Scholar

  • Меррилл, Р.Т. и П. Л. Макфадден, Стабильность геомагнитного поля: события разворота и экскурсии, Планета Земля. Sci. Lett. , 121 , 57–69, 1994.

    Артикул Google Scholar

  • Милановский Э. Э., Неотектоника Кавказа , 278 с., Недра, 1978.

    Google Scholar

  • Мотидзуки Н., Х. Цунакава, Х. Сибуя, Дж. Кэссиди и И.Смит Э. М., Палеонапряженности геомагнитных экскурсий Окленда по методу Шоу LTD-DHT, Phys. Планета Земля. Интер. , 154 , 168–179, 2006.

    Статья Google Scholar

  • Мотидзуки, Н., Х. Цунакава, Х. Сибуя, Т. Тагами, А. Одзава и И.Э.М. Смит, Дальнейшее K-Ar датирование и палеомагнитное исследование геомагнитных экскурсий в Окленде, Earth Planets Space , 59 , 755–761, 2007.

    Артикул Google Scholar

  • Нагата Т., Р. М. Фишер и К. Момосе, Вековые вариации полной геомагнитной силы за последние 5000 лет, J. Geophys. Res. , 68 , 5277–5281, 1963.

    Статья Google Scholar

  • Оздемир О., Инверсия титаномагемитов, Phys. Планета Земля. Интер. , 65 , 125–136, 1987.

    Артикул Google Scholar

  • Петрониль, М., А. Гогуитчайшвили, Б. Генри, Л. Альва-Вальдивия, Х. Росас-Эльгера, М. Родригес Сеха и М. Кальво-Ратерт, Палеомагнетизм Ар-Ар датировал потоки лавы из вулканическое поле Себоруко-Сан-Педро (западная Мексика): свидетельство предвестника перехода Матуяма-Брюнес и полностью обращенного геомагнитного события в хронике Брюнеса, J. Geophys. Res. , 110 , B08101, DOI: 10.1029 / 2004jb003321, 2005.

  • Прево М., Р.С. Майнкинен, С. Громме и А. Лекай, Высокая палеонапряженность геомагнитного поля по результатам термомагнитных исследований подушечных базальтов рифтовой долины Среднего Атлантического хребта, J. Geophys. Res. , 88 , 2316–2326, 1983.

    Артикул Google Scholar

  • Прево М., Р. С. Майнкинен, Р. С. Коу и С. Громме, Переход геомагнитной полярности горы Стинс (Орегон) 2.Вариации напряженности поля и обсуждение моделей инверсии, J. Geophys. Res. , 90 , 10417–10448, 1985.

    Артикул Google Scholar

  • Quidelleur, X., J. Carlut, P. Y. Gillot, V. Soler, Эволюция геомагнитного поля до перехода Матуяма-Брюнес: радиометрическое датирование экскурсии 820 тыс. Лет назад на Ла-Пальма, Geophys. J. Int. , 151 , F6 – F10, 2002.

    Артикул Google Scholar

  • Риисагер, П.и Н. Абрахамсен, Палеонапряженность палеоценовых базальтов Западной Гренландии: асимметричная интенсивность вокруг перехода C27n-C26r, Phys. Планета Земля. Интер. , 118 , 53–64, 2000.

    Артикул Google Scholar

  • Риисагер П., Дж. Риисагер, Н. Абрахамсен и Р. Ваагштейн, Эксперименты по палеинтенсивности Телье на базальтах Фарерских островов: технические аспекты и геомагнитные последствия, Phys. Планета Земля.Интер. , 131 , 91–100, 2002.

    Артикул Google Scholar

  • Сингер, Б.С., М.К. Релле, К.А. Хоффман, А. Баттл, К. Ладж, Х. Гийу и Дж. Карраседо, Ar / Ar возрасты из переходно намагниченных лав на Ла-Пальма, Канарские острова, и геомагнитная нестабильность шкала времени, J. Geophys. Res. , 107 (B11), 10.1029 / 2001JB001613, 2002.

  • Сологашвили Дж. Палеомагнетизм неогеновых вулканических образований Грузии: Дис., 1986.

    Google Scholar

  • Танака, Х., М. Коно и С. Канеко, Палеосекулярные вариации направления и интенсивности из двух плиоцен-плейстоценовых разрезов лавы на юго-западе Исландии, J. Geomag. Геоэлектр. , 47 , 89–102, 1995.

    Статья Google Scholar

  • Tauxe, L., Отложения относительной палеонапряженности: теория и практика, Rev.Geophys. , 31 , 319–354, 1993.

    Артикул Google Scholar

  • Tauxe, L., T. A. T. Mullender и T. Pick, Pot-belies, sys-Waists and superparamagnetism in Magnetic Hysteresis, J. Geophys. Res. , 95 , 12337–12350, 1996.

    Артикул Google Scholar

  • Телье, Э. и О. Телье, Recherches géomagnetiques sur les coulees volcaniques d’Auvergne, Ann.Geophys. , 1 , 37–52, 1944.

    Google Scholar

  • Телье, Э. и О. Телье, Sur l’intensité du champ magnétique terrestre dans le passé Historique et géologique, Ann. Геофис. , 15 , 285–376, 1959.

    Google Scholar

  • Валет, Дж. П. и Э. Эрреро-Бервера, Эксперименты по палеонапряженности с использованием размагничивания переменного поля, Планета Земля.Sci. Lett. , 177 , 43–58, 2000.

    Артикул Google Scholar

  • Валет, Дж. П. и Э. Эрреро-Бервера, Некоторые характеристики геомагнитных инверсий, выведенные из подробных вулканических записей, C. R. Geosci. , 335 , 79–90, 2003.

    Артикул Google Scholar

  • Обратная матрица: метод исключения Гаусса

    Вычисление обратной матрицы – незаменимый инструмент линейной алгебры.{-1} = I $$$

    , где $$ I $$ – единичная матрица, все элементы которой равны нулю, за исключением элементов на главной диагонали, которые равны $$ 1 $$.

    Пусть будет матрица:

    $$$ A = \ left (\ begin {array} {ccc} 1 & 1 & 0 \\ 1 & 0 & 1 \\ 0 & 1 & 0 \ end {array} \ справа) $$$

    Как найти обратную матрицу, используя метод исключения Гаусса?

    1) Единичная матрица добавляется к матрице $$ A $$.

    $$$ \ left (\ begin {array} {cccccc} 1 & 1 & 0 & 1 & 0 & 0 \\ 1 & 0 & 1 & 0 & 1 & 0 \\ 0 & 1 & 0 & 0 & 0 & 1 \ end {array} \ right) $$$

    (Обратите внимание, что $$ I = \ left (\ begin {array} {ccc} 1 & 0 & 0 \\ 0 & 1 & 0 \\ 0 & 0 & 1 \ end {array} \ right) $$)

    2) С помощью метода Гаусса мы попытаемся передать единичную матрицу в левую часть.{-1} $$.

    Что является инверсией следующей матрицы?

    $$$ A = \ left (\ begin {array} {ccc} 1 & 1 & 0 \\ 1 & 0 & 1 \\ 0 & 1 & 0 \ end {array} \ right) $$$

    Мы должны следовать процедуре шаг за шагом.

    1) Прежде всего, единичная матрица добавляется справа от исходной матрицы:

    $$$ \ left (\ begin {array} {cccccc} 1 & 1 & 0 & 1 & 0 & 0 \\ 1 & 0 & 1 & 0 & 1 & 0 \\ 0 & 1 & 0 & 0 & 0 & 1 \ end {array} \ right) $$$

    2) Мы должны “сдвинуть” единичную матрицу влево с помощью метода Гаусса.

    Этот метод требует некоторой интуиции, поскольку он не является точным руководством. В любом случае интуицию можно заменить практикой, и метод Гаусса оказывается намного проще, чем кажется на первый взгляд.

    $$$ \ left (\ begin {array} {cccccc} 1 & 1 & 0 & 1 & 0 & 0 \\ 1 & 0 & 1 & 0 & 1 & 0 \\ 0 & 1 & 0 & 0 & 0 & 1 \ end {array} \ right) \ rightarrow (row2-row1) \ rightarrow \ left (\ begin {array} {cccccc} 1 & 1 & 0 & 1 & 0 & 0 \\ 0 & -1 & 1 & -1 & 1 & 0 \\ 0 & 1 & 0 & 0 & 0 & 1 \ end {array} \ right) $$$

    $$$ \ left (\ begin {array} {cccccc} 1 & 1 & 0 & 1 & 0 & 0 \\ 0 & -1 & 1 & -1 & 1 & 0 \\ 0 & 1 & 0 & 0 & 0 & 1 \ end {array} \ right) \ rightarrow (row3 + row2 ) \ rightarrow \ left (\ begin {array} {cccccc} 1 & 1 & 0 & 1 & 0 & 0 \\ 0 & -1 & 1 & -1 & 1 & 0 \\ 0 & 0 & 1 & -1 & 1 & 1 \ end {array} \ right) $$$

    $$$ \ left (\ begin {array} {cccccc} 1 & 1 & 0 & 1 & 0 & 0 \\ 0 & -1 & 1 & -1 & 1 & 0 \\ 0 & 0 & 1 & -1 & 1 & 1 \ end {array} \ right) \ rightarrow (row2-row3) \ rightarrow \ left (\ begin {array} {cccccc} 1 & 1 & 0 & 1 & 0 & 0 \\ 0 & -1 & 0 & 0 & 0 & -1 \\ 0 & 0 & 1 & -1 & 1 & 1 \ end {array} \ right) $$$

    $$$ \ left (\ begin {array} {cccccc} 1 & 1 & 0 & 1 & 0 & 0 \\ 0 & -1 & 0 & 0 & 0 & -1 \\ 0 & 0 & 1 & -1 & 1 & 1 \ end {array} \ right) \ rightarrow (row1 + row2) \ rightarrow \ left (\ begin {array} {cccccc} 1 & 0 & 0 & 1 & 0 & -1 \\ 0 & -1 & 0 & 0 & 0 & -1 \\ 0 & 0 & 1 & -1 & 1 & 1 \ end {array} \ right) $$$

    Наконец, $$ row2 $$ умножается на $$ (- 1 ) $$, и поэтому мы имеем идентичность слева.

    Оставить комментарий